Закрыть

Формула емкости конденсатора: Формула емкости конденсатора, С

Содержание

Формула емкости конденсатора, С

Если q – величина заряда одной из обкладок конденсатора, а – разность потенциалов между его обкладками, то величина C, равная:

   

называется емкостью конденсатора. Это постоянная величина, которая зависит то размеров и устройства конденсатора.

Рассмотрим два одинаковых конденсатора, разница между которым заключается только в том, что между обкладками одного вакуум (или часто говорят воздух), между обкладками другого находится диэлектрик. В таком случае при равных зарядах на конденсаторах разность потенциалов воздушного конденсатора будет в раз меньше, чем между обкладками второго. Значит емкость конденсатора с диэлектриком (C) в раз больше, чем воздушного ():

   

где – диэлектрическая проницаемость диэлектрика.

За единицу емкости конденсатора принимают емкость такого конденсатора, который единичным зарядом (1 Кл) заряжается до разности потенциалов, равной одному вольту (в СИ).

Единицей емкости конденсатора (как и любой эклектической емкости) в международной системе единиц (СИ) служит фарад (Ф).

Формула электрической емкости плоского конденсатора

Поле между обкладками плоского конденсатора обычно считают однородным. Его однородность нарушается только около краев. При вычислении емкости плоского конденсатора этими краевыми эффектами часто пренебрегают. Это следует делать, если расстояние между пластинами мало в сравнении с их линейными размерами. Для расчета емкости плоского конденсатора применяют формулу:

   

где – электрическая постоянная; S – площадь каждой (или наименьшей) пластины; d – расстояние между пластинами.

Электрическая емкость плоского конденсатора, который содержит N слоев диэлектрика толщина каждого , соответствующая диэлектрическая проницаемость i-го слоя , равна:

   

Формула электрической емкости цилиндрического конденсатора

Цилиндрический конденсатор представляется собой две соосных (коаксиальных) цилиндрические проводящие поверхности, разного радиуса, пространство между которыми заполняет диэлектрик.

Электрическая емкость цилиндрического конденсатора вычисляется как:

   

где l – высота цилиндров; – радиус внешней обкладки; – радиус внутренней обкладки.

Формула электрической емкости сферического конденсатора

Сферическим конденсатором называют конденсатор, обкладками которого являются две концентрические сферические проводящие поверхности, пространство между ними заполнено диэлектриком. Емкость такого конденсатора находят как:

   

где – радиусы обкладок конденсатора.

Примеры решения задач по теме «Емкость конденсатора»

По какой формуле найти ёмкость (объем) конденсаторов

Во всех электронных устройствах используются конденсаторы. При их конструировании или изготовлении своими руками параметры устройств рассчитываются по специальным формулам.

Конденсаторы

Расчёт конденсаторов

Один из главных параметров таких устройств – ёмкость. Рассчитать её можно по следующей формуле:

C=q/U, где:

  • C – ёмкость,
  • q – заряд одной из обкладок элемента,
  • U – разность потенциалов между обкладками.

В электротехнике вместо понятия «разность потенциалов между обкладками» используется «напряжение на конденсаторе».

Ёмкость элемента не зависит от конструкции и размеров устройства, а только от напряжения на нём и заряда обкладок. Но эти параметры могут изменяться в зависимости от расстояния между ними и материала диэлектрика. Это учитывается в формуле:

С=Co*ε, где:

  • С – реальная ёмкость,
  • Со – идеальная, при условии, что между пластинами вакуум или воздух,
  • ε – диэлектрическая проницаемость материала между ними.

Например, если в качестве диэлектрика используется слюда, «ε» которой 6, то ёмкость такого устройства в 6 раз больше, чем воздушного, а при изменении количества диэлектрика меняются параметры конструкции. На этом принципе основана работа ёмкостного датчика положения.

Устройство конденсатора

Единицей ёмкости в системе СИ является 1 фарад (F). Это большая величина, поэтому чаще применяются микрофарады (1000000mkF=1F) и пикофарады (1000000pF=1mkF).

Расчет плоской конструкции

Если нужно рассчитать плоский конденсатор, то необходимо учесть площадь обкладок и расстояние между ними. Это отражено в формуле, по которой рассчитывается ёмкость плоского конденсатора:

C=ε/d, где:

  • ε – диэлектрическая проницаемость изолирующего материала,
  • d – расстояние между пластинами.

Расчет конструкции цилиндрической формы

Цилиндрический конденсатор – это две соосные трубки различного диаметра, вставленные друг в друга. Между ними находится диэлектрик. При радиусе цилиндров, близком друг к другу и намного большем, чем расстояние между ними, цилиндрической формой можно пренебречь и свести расчёт к формуле, аналогичной той, по которой рассчитывается плоский конденсатор.

Вычисляются параметры такого устройства по формуле:

C=(2π*l*R*ε)/d, где:

  • l – длина устройства,
  • R – радиус цилиндра,
  • ε – диэлектрическая проницаемость изолятора,
  • d – его толщина.

Расчёт сферической конструкции

Есть устройства, обкладки которых представляют собой два шара, вложенные друг в друга. Формула ёмкости такого прибора:

C=(4π*l*R1*R2*ε)/(R2-R1), где:

  • R1 – радиус внутренней сферы,
  • R2 – радиус внешней сферы,
  • ε – диэлектрическая проницаемость.

Формулы ёмкости конденсаторов различной формы

Ёмкость одиночного проводника

Кроме конденсаторов, способностью накапливать заряд обладают отдельные проводники. Одиночным проводником считается такой проводник, который бесконечно далёк от других проводников. Параметры заряженного элемента рассчитывается по формуле:

C=Q/φ, где:

  • Q – заряд,
  • φ – потенциал проводника.

Объём заряда определяется размером и формой устройства, а также окружающей средой. Материал прибора значения не имеет.

Способы соединения элементов

Не всегда есть в наличии элементы с необходимыми параметрами. Приходится соединять их различными способами.

Соединение конденсаторов

Параллельное соединение

Это такое соединение деталей, при котором к одной клемме или контакту присоединяются первые обкладки каждого конденсатора. При этом вторые обкладки присоединяются к другой клемме.

При таком соединении напряжение на контактах всех элементов будет одинаковым. Заряд каждого из них происходит независимо от остальных, поэтому общая ёмкость равна сумме всех величин. Её находят по формуле:

C=C1+C2+…Cn,

где C1-Cn – параметры деталей, участвующих в параллельном соединении.

Важно! Конденсаторы имеют предельное допустимое напряжение, превышение которого приведёт к выходу элемента из строя. При параллельном соединении устройств с различным допустимым напряжением этот параметр получившейся сборки равен элементу с наименьшим значением.

Последовательное соединение

Это такое соединение, при котором к клемме присоединяется только одна пластина первого элемента. Вторая пластина присоединяется к первой пластине второго элемента, вторая пластина второго – к первой пластине третьего и так далее. Ко второй клемме присоединяется только вторая обкладка последнего элемента.

При таком соединении заряд на обкладках конденсатора в каждом приборе будет равен остальным, однако напряжение на них будет разным: для зарядки устройств большей ёмкости тем же зарядом требуется меньшая разность потенциалов. Поэтому вся цепочка представляет собой одну конструкцию, разность потенциалов которой равна сумме напряжений на всех элементах, а заряд конденсатора равен сумме зарядов.

Последовательное соединение увеличивает допустимое напряжение и уменьшает общую ёмкость, которая меньше самого меньшего элемента.

Рассчитываются эти параметры следующим образом:

  • Допустимое напряжение:

Uобщ=U1+U2+U3+…Un, где U1-Un – напряжение на конденсаторе;

  • Общая ёмкость:

1/Собщ=1/С1+1/С2+1/С3+…1/Сn, где С1-Сn – параметры каждого устройства.

Интересно. Если в цепи только два элемента, то можно воспользоваться упрощённой формулой: Собщ=(С1*С2)/(С1+С2).

Смешанное соединение

Это такое соединение, в котором есть детали, соединённые последовательно, и есть соединённые параллельно. Параметры всей цепи рассчитывается в следующей последовательности:

  1. определяются группы элементов, соединённые параллельно;
  2. для каждой группы в отдельности рассчитывается эквивалентные значения;
  3. рядом с каждой группой параллельно соединённых деталей пишутся получившиеся величины;
  4. получившаяся схема эквивалентна последовательной схеме и рассчитывается по соответствующим формулам.

Знание формул, по которым можно найти емкость при изготовлении конденсаторов или их соединении необходимо при конструировании электронных схем.

Видео

Оцените статью:

Глава 20.

Конденсаторы

Для накопления разноименных электрических зарядов служит устройство, которое называется конденсатором. Конденсатор — система двух изолированных друг от друга проводников (которые часто называют обкладками конденсатора), один из которых заряжен положительным, второй — таким же по величине, но отрицательным зарядом. Если эти проводники представляют собой плоские параллельные пластинки, расположенные на небольшом рас-стоянии друг от друга, то конденсатор называется плоским.

Для характеристики способности конденсатора накапливать заряд вводится понятие электроемкости (часто говорят просто емкости). Емкостью конденсатора называется отношение заряда конденсатора к той разности потенциалов , которая возникает между обкладками при их заряжении зарядами и (эту разность потенциалов проводников часто называют электрическим напряжением между обкладками и обозначают буквой ):

(20.1)

Поскольку величины и (или ) в формуле (20. 1) зависимы, то емкость (20.1) не зависит от и , а является характеристикой геометрии системы проводников. Действительно, при сообщении проводникам зарядов и проводники приобретут потенциалы, разность которых будет пропорциональна заряду . Поэтому в отношении (20.1) заряд сокращается.

Выведем формулу для емкости плоского конденсатора (эта формула входит в программу школьного курса физики). При заряжении параллельных пластин, расположенных на небольшом расстоянии друг от друга, зарядами и , в пространстве между ними возникает однородное электрическое поле с напряженностью (см. гл. 18):

(20.2)

Разность потенциалов между пластинами равна

(20.3)

где — площадь пластин, — расстояние между ними. Отсюда, вычисляя отношение заряда к разности потенциалов (20. 3), находим емкость плоского конденсатора

(20.4)

Если все пространство между обкладками заполнено диэлектриком с диэлектрической проницаемостью , то поле (20.2) и разность потенциалов (20.3) убывает в раз, а емкость конденсатора в раз взрастает

(20.5)

Для конденсаторов, соединенных в батареи, вводится понятие эквивалентной емкости, как емкости одного конденсатора, который при заряжении его тем же зарядом, что и батарея дает ту же разность потенциалов, что и батарея конденсаторов. Приведем формулы для эквивалентной емкости, а также для заряда и электрического напряжения на каждом конденсаторе при последовательном и параллельном их соединении.

Последовательное соединение (см. рисунок). При сообщении левой пластине левого конденсатора заряда , а правой пластине правого заряда , на внутренних пластинах благодаря поляризации будут индуцироваться заряды (см.

рисунок; значения индуцированных зарядов приведены под пластинами). Можно доказать, что в результате поляризации каждый конденсатор будет заряжен такими же зарядами и , как и заряды крайних пластин, напряжение на всей батарее конденсаторов равно сумме напряжений на каждом, а обратная эквивалентная емкость батареи — сумме обратных емкостей всех конденсаторов

(20.6)

Параллельное соединение (см. рисунок). В этом случае если сообщить левому проводнику заряд , правому сообщить заряд , заряд распределится между конденсаторами, вообще говоря, не одинаково, но по закону сохранения заряда .

Поскольку правые пластины всех конденсаторов соединены между собой, левые — тоже, то они представляют собой единые проводники, и, следовательно, разность потенциалов между пластинами каждого конденсатора будет одинакова: . Можно доказать, что при таком соединении конденсаторов эквивалентная емкость батареи равна сумме емкостей отдельных конденсаторов

(20. 7)

Заряженный конденсатор обладает определенной энергией. Если конденсатор емкости заряжен зарядом , то энергия этого конденсатора (можно говорить энергия электрического поля конденсатора) равна

(20.8)

С помощью определения электрической емкости (20.1) можно переписать формулу (20.8) еще в двух формах:

(20.9)

Рассмотрим в рамках этого минимума сведений о конденсаторах типичные задачи ЕГЭ по физике, которые были предложены в первой части книги.

Электроемкость конденсатора — его геометрическая характеристика, которая при неизменной геометрии не зависит от заряда конденсатора (задача 20.1.1 — ответ 3). Аналогично не меняется емкость конденсатора при увеличении напряжения на конденсаторе (задача 20.1.2 — ответ 3).

Связь между единицами измерений (задача 20.1.3) следует из определения емкости (20.1). Единица электрической емкости в международной системе единиц измерений СИ называется Фарада. 1 Фарада — это емкость такого конденсатора, между пластинами которого возникает напряжение 1 В при зарядах пластин 1 Кл и -1 Кл (ответ 4).

Поскольку электрическое поле в плоском конденсаторе однородно, то напряженность поля в конденсаторе и напряжение между пластинами связаны соотношением (см. формулу (18.9)) , где — расстояние между пластинами. Отсюда находим напряженность поля между обкладками плоского конденсатора в

задаче 20.1.4

(ответ 4).

Согласно определению электрической емкости имеем в задаче 20.1.5

(ответ 2).

Из формулы (20.4) для емкости плоского конденсатора заключаем, что при увеличении площади его пластин в 3 раза (задача 20. 1.6) его емкость увеличивается в 3 раза (ответ 1).

При уменьшении в раз расстояния между пластинами емкость плоского конденсатора возрастет в раз. Поэтому новое напряжение на конденсаторе (задача 20.1.7) можно найти из следующей цепочки формул

где и — новый заряд конденсатора (ответ 3).

Так как конденсатор в задаче 20.1.8 подключен к источнику, то между его пластинами поддерживается постоянное напряжение независимо от расстояния между ними. Поэтому заряд конденсатора изменяется при раздвигании пластин так же, как изменяется его емкость. А поскольку при увеличении расстояния между пластинами вдвое емкость конденсатора уменьшается вдвое (см. формулу (20.4)), то вдвое уменьшается и заряд конденсатора (ответ 2).

В задаче 20.1.9 конденсатор отключен от источника в процессе сближения пластин. Поэтому не меняется их заряд. А поскольку напряженность электрического поля между пластинами определяется соотношением (20. 2)

то напряженность электрического поля между пластинами также не изменяется (ответ 3). Этот же результат можно получить и через определение емкости с учетом того, что

произведение от расстояния между пластинами не зависит (см. формулу (20.4)).

Из формул (20.8), (20.9) видим, что только одно из приведенных в качестве ответов к задаче 20.1.10 соотношений (а именно — 2) определяет энергию конденсатора.

При последовательном соединении конденсаторов (задача 20.2.1) одинаковыми будут их заряды независимо от значений их электрических емкостей (ответ 2). При параллельном соединении конденсаторов (задача 20.2.2) одинаковыми будут напряжения на каждом из них (ответ 3).

Поскольку конденсатор в задаче 20.2.3 отключен от источ-ника напряжения, его заряд не меняется в процессе раздвигания пластин. Поэтому для исследования изменения энергии конденсатора удобно воспользоваться формулой (20. 8)

(1)

Так как при увеличении расстояния между пластинами в раз электрическая емкость конденсатора уменьшается в раз, то согласно формуле (1) энергия конденсатора увеличится в раз (ответ 1).

В задаче 20.2.4 не изменяется напряжение на конденсаторе. Поэтому воспользуемся первой из формул (20.9)

Из этой формулы заключаем, что при увеличении в раз расстояния между пластинами энергия конденсатора уменьшится в раз — ответ 2. (Разница с предыдущей задачей связана с тем, что здесь кроме внешних сил, совершающих работу при раздвигании пластин, совершает работу источник напряжения.)

В задаче 20.2.5 изменяют расстояние между пластинами (и, следовательно, емкость) и заряд конденсатора. Поэтому удобно воспользоваться формулой (20.8)

Из этой формулы заключаем, что при увеличении расстояния между пластинами в 2 раза и увеличении заряда конденсатора в 2 раза его энергия возрастет в 8 раз (ответ 4).

Поскольку в задаче 20.2.6 конденсаторы соединены последовательно, емкость батареи конденсаторов можно найти по формуле (20.6), откуда находим емкость батареи конденсаторов (ответ 2).

В задаче 20.2.7 конденсаторы соединены параллельно, поэтому емкость батареи конденсаторов можно найти по формуле (20.7): (ответ 2).

Основной вопрос, на который нужно ответить в задаче 20.2.8, это как соединены конденсаторы? Последовательно, параллельно, по-другому? Попробуем по-другому расположить в пространстве и изменить длину соединительных проводов, чтобы схема стала более понятной. Очевидно, что можно соединить вершину 1 и вершину 3 («уменьшив» длину провода 1-3), а также вершины 2 и 4. При этом средний конденсатор разворачивается в пространстве, и схема приобретает вид, показанный на рисунке, откуда видно, что конденсаторы соединены параллельно. Поэтому (ответ 1).

Когда в заряженный плоский конденсатор вставляют металлическую пластинку (задача 20. 2.9), параллельную обкладкам конденсатора, напряженность электрического поля внутри пластинки становится равным нулю, вне пластинки между обкладками конденсатора остается таким же, каким оно было в отсутствие пластинки , где — заряд конденсатора, — площадь его пластин. Поэтому напряжение между обкладками конденсатора определяется соотношением:

где — расстояние между обкладками конденсатора, — толщина пластинки. Отсюда находим емкость рассматриваемого конденсатора

(ответ 4).

Чтобы найти емкость сферического конденсатора (задача 20.2.10) сообщим его обкладкам заряды и , найдем напряжение между обкладками, вычислим отношение заряда к напряжению. Разность потенциалов двух концентрических сфер, заряженных зарядами и (напряжение между обкладками сферического конденсатора), определена в задаче 19.2.5., откуда находим электрическую емкость сферического конденсатора (ответ 3):

Нахождение емкости конденсатора.

Наука техника технологии

Плоский конденсатор состоит из двух параллельных пластин, разделённых небольшим зазором шириной , заполненным однородным диэлектриком.

Нам известно, что поле между двумя разноимённо заряженными пластинами с одинаковой по величине поверхностной плотностью равно, где,S– площадь каждой пластины. Напряжение между обкладками:

Используя определение емкости конденсатора, получаем:

Отметим, что полученная формула является приближенной, так как выведена без учета искажения поля у краев пластин. Расчет по этой формуле дает завышенное значение ёмкости и тем точнее, чем меньше зазор по сравнению с линейными размерами пластин.

Ёмкость сферического конденсатора.

Сферический конденсатор представляет собой систему двух концентрических сфер с радиусами и. Электрическое поле между обкладками сферического конденсатора согласно теореме Гаусса определяется зарядом внутренней сферы. Напряжение между обкладками равно:

.

Для ёмкости сферического конденсатора получаем:

Это формула точная.

Если , полученная формула переходит в выражение для ёмкости плоского конденсатора.

Ёмкость цилиндрического конденсатора.

Цилиндрический конденсатор составляет систему двух коаксиальных цилиндров с радиусами и, длиной.

Рассуждая аналогично выводу ёмкости сферического конденсатора, получаем:

..

Полученная формула является приближенной и при малом зазоре переходит в формулу емкости плоского конденсатора.

Соединение конденсаторов.

В практике для получения необходимых значений емкости используют соединения конденсаторов: а) последовательное, б) параллельное, в) смешанное (см. рисунок).


Ёмкость последовательного соединения конденсаторов.

Заряды последовательно соединенных конденсаторов равны , а напряжение на батарее. Из определения емкости следует:

Если , то(ёмкость последовательного соединения меньше наименьшей ёмкости в последовательном соединении).

Для последовательно соединенных конденсаторов емкость вычисляется по формуле:

Ёмкость параллельного соединения конденсаторов.

Заряд батареи равен сумме зарядов:

а напряжение . По определению емкости получаем:

Для параллельно соединенных конденсаторов:.

В случае одинаковых конденсаторов: .

Оценить емкость батареи (см. рисунок) .

Используя свойство бесконечности можно представить цепь в виде соединения (см. рисунок).

Для расчета ёмкости батареи получаем:

Откуда: , так как, то.

Лекция 7.

Диэлектрики в электрическом поле.

Диэлектриками (изоляторами) называют вещества, не проводящие постоянного электрического тока. Это означает, что в диэлектриках отсутствуют «свободные» заряды, способные перемещаться на значительные расстояния.

Диэлектрики состоят либо из нейтральных молекул, либо из ионов, находящихся в узлах кристаллической решетки. Сами же молекулы могут быть полярными инеполярными. Полярные молекулы обладают дипольным моментом, у неполярных молекул дипольный момент равен нулю.

Поляризация.

В электрическом поле диэлектрики поляризуются. Это явление связано с появлением в объеме и на поверхности диэлектрика «связанных » зарядов. При этом конечный объем диэлектрика приобретает дипольный момент. Механизм поляризации связан с конкретным строением диэлектрика. Если диэлектрик состоит из неполярных молекул, то в пределах каждой молекулы происходит смещение зарядов – положительных по полю, отрицательных против поля, т.е. молекулы, приобретают дипольный момент. У диэлектрика с полярными молекулами в отсутствии внешнего электрического поля их дипольные моменты ориентированы хаотично.

Под действием электрического поля диполи ориентируются преимущественно в направлении поля. Рассмотрим подробнее этот механизм (см. рисунок). Пара сил исоздает вращательный момент равный, где- дипольный момент молекулы. Этот момент стремится ориентировать диполь вдоль поля. В ионных кристаллах под действием электрического поля все положительные ионы смещаются по полю, отрицательные – против поля. Отметим, что смещение зарядов очень малы даже по сравнению с размерами молекул. Это связано с тем, что напряженность внешнего электрического поля обычно много меньше напряженности внутренних электрических полей в молекулах.

Отметим, что существуют диэлектрики, поляризованные даже при отсутствии внешнего поля (электреты, сегнетоэлектрики). Мы остановимся на рассмотрении только однородных диэлектриков, в которых отсутствует остаточная поляризация, а объемный и «связанный» заряд всегда равен нулю .

Плоский конденсатор – это физическое упрощение, взявшее начало из ранних исследований электричества, представляющее собой конструкцию, где обкладки имеют форму плоскостей и в каждой точке параллельны.

Формулы

Многие ищут формулы, описывающие ёмкость плоского конденсатора. Если это так, то не читайте ниже любопытные и малоизвестные факты, потому что сухие математические знаки, конечно же, важнее.

Первым определил ёмкость плоского конденсатора Вольта. В его распоряжении ещё не было такой величины, как разница потенциалов, именуемая напряжением, но интуитивно он совершенно правильно объяснил суть явления. Что касается количества зарядов, то он трактовал её, как объем электрического флюида атмосферы – не совсем правильно, но в конечном итоге похоже на правду. Согласно этому мировоззрению ёмкость плоского конденсатора может быть найдена, как отношение объёма накопленного электрического флюида к разнице атмосферных потенциалов, то есть:

Эта формула применима к любому конденсатору, вне зависимости от его конструкции. То есть, является универсальной. Специально для плоских конденсаторов имеется формула ёмкости, выраженная через свойства материала диэлектрика и геометрические размеры:

В этой формуле через S обозначена площадь обкладок, вычисляемая через произведение сторон, а d – показывает расстояние между обкладками. Прочие символы – электрическая постоянная (8,854 пФ/м) и диэлектрическая проницаемость материала диэлектрика, да простит Тот столь откровенные тавтологии. Электролитические конденсаторы обладают столь большой ёмкостью по той причине, что проводящий раствор отделен от металла очень тонким слоем оксида. Следовательно, d в этом случает будет минимальным. Единственный минус в том, что электролитические конденсаторы полярные, их нельзя подключать в цепи переменного тока. С этой целью на каждом анод или катод обозначены значками плюса или минуса.

Плоские конденсаторы сегодня редко встречаются, и это преимущественно плёночные микроскопические технологии, где такой род поверхностей является доминирующим. Все пассивные и активные элементы образуются через трафарет. И, следовательно, имеют вид плёнок. Плоские индуктивности, резисторы и конденсаторы наносятся в виде токопроводящих паст.

От материала диэлектрика ёмкость зависит по той причине, что у каждого из них структура своя. Считается, что аморфное вещество состоит из неориентированных диполей, упруго укреплённых на своих местах. При приложении внешнего электрического поля они обратимо ориентируются вдоль силовых линий, ослабляя напряжённость. В результате заряд продолжает накапливаться, пока этот процесс не прекратится. По мере выхода энергии из обкладок диполи возвращаются на свои места, делая возможным следующий рабочий цикл. Так функционирует плоский электрический конденсатор.


Из истории

Исторически первым начал исследовать накопление заряда великий Алессандро Вольта. В докладе Королевскому научному обществу за 1782 год он впервые озвучил слово конденсатор. В понимании Вольты электрофорус, представляющий собой две параллельные обкладки, выкачивал из эфира электрический флюид.

В то время все познания сводились к тому, что учёные думали, будто атмосфера Земли содержит в себе нечто, что не может быть определено приборами. Существовали только простейшие электроскопы, способные определить знак заряда и его наличие, но не дававшие представления о количестве. Учёные просто натирали мехом поверхность тела и подносили его для исследования в область влияния прибора. Ещё Гильберт показал, что электрические и магнитные взаимодействия ослабевают с расстоянием. Поэтому учёные примерно знали, что нужно делать, но исследования не продвигались ни на йоту.

Гипотеза об атмосферном электричестве высказана Бенджамином Франклином. Он активно исследовал молнии и пришёл к выводу, что это проявления все той же единой силы. Запуская воздушного змея в небо, он соединял его шёлковой нитью с землёй и наблюдал дуговой разряд. Это были достаточно опасные опыты, и Бенджамин много раз рисковал своей жизнью ради развития науки. О том, что шёлковая нить проводит статический заряд, было известно от Стивена Грея, первым собравшего в 1732 году электрическую цепь.

Уже через 20 лет (1752 год) Бенджамин Франклин предложил конструкцию первого громоотвода, осуществлявшего молниезащиты близлежащих построек. Только вдуматься! – до этого всякий мог ожидать того, что его дом сгорит от случайного удара. Именно Бенджамин Франклин предложил один из видов заряда называть положительным (стеклянный), а другой отрицательным (смоляной). Так физики были введены в заблуждение относительно истинного направления движения электронов. Но как они могли думать иначе, когда в 1802 году на примере опытов нашего соотечественника Петрова увидели, что на аноде образуется ямка? Следовательно, положительные частицы переносили заряд на катод, вот только это были ионы воздушной плазмы.

К началу исследования Вольтой электрических явлений, таким образом, были уже известны статические заряды и факт наличия у них двух знаков, кроме того люди упорно считали, что весь «флюид» берётся из воздуха. На эту мысль их натолкнули опыты с натиранием янтаря шерстью, которые не могли быть проведены под водой. Следовательно, логичным было предположить, что электричество может происходить только из атмосферы Земли, что, конечно же, совершенно неверно. В частности, многие растворы, исследованные Хампфри Дэви, могут проводить электрический ток.

Причина, следовательно, была в другом – при натирании янтаря под водой силы трения снижались в десятки и сотни раз, а заряд рассеивался по всему объёму жидкости. Следовательно, этот процесс был всего лишь неэффективным. Но сегодня каждый добытчик знает, что нефть прекрасно электризуется трением о трубы и без воздуха. Следовательно, атмосфера для «флюида» не является обязательным компонентом.

Самый большой в мире плоский конденсатор

Столь систематизированные, но в корне неверные толкования все-таки не смогли остановить Вольту на его исследовательском пути. Он упорно изучал электрофорус, как один из самых совершенных генераторов, существовавших в то время. Вторым был серный шар Отто фон Герике, изобретённый более чем за век до этого (1663 год). С тех самых пор его конструкция мало менялась, но после открытий Стивена Грея заряд начали снимать при помощи проводников. В частности, в для этого служат металлические гребёнки-нейтрализаторы.

Долгое время учёные ходили вокруг да около. Электрофорная машина 1880 года может считаться первым мощным генератором разряда, позволявшим получить дугу, но своей настоящей силы электроны достигли в генераторе Ван де Граафа (1929 год), где разница потенциалов составила единицы мегавольта. Для сравнения грозовое облако, согласно данным Википедии, может иметь потенциал относительно Земли в единицы гигавольт (на три порядка больше, чем в человеческой машине).

Суммируя сказанное, можно с определённой долей уверенности сказать, что природные процессы используют в качестве принципа своего действия электризацию трением, влиянием и некоторые другие её виды, а мощный циклон является самым большим из известных нам плоских конденсаторов. Молния показывает, что бывает, когда диэлектрик (атмосфера) не выдерживает приложенной разницы потенциалов и пробивается. В точности то же самое происходит в любом плоском конденсаторе, созданном человеком, если вольтаж оказывается для него непомерным. Пробой твёрдого диэлектрика необратим, а возникающая электрическая дуга часто служит причиной расплавления обкладок и выхода изделия из строя.

Электрофорус

Итак, Вольта взялся за исследование модели природных процессов. Первый электрофорус появился в 1762 году сконструированный Йоханом Карлом Вильке. По-настоящему популярным прибор становится после докладов Вольты Королевскому научному обществу (середина 70-х годов XVIII века). Вольта же и дал прибору его нынешнее название.


Электрофорус способен накапливать электростатический заряд, образованный трением резины куском шерсти. Он состоит из двух плоских, параллельных друг другу обкладок:

  • Нижняя представляет собой тонкий кусок резины. Толщина его выбирается из соображений эффективности устройства. Если выбрать кусок более солидный, то значительная часть энергии будет накапливаться внутри диэлектрика на ориентацию его молекул. Что и наблюдается в современном плоском конденсаторе, куда диэлектрик помещается для увеличения электроёмкости.
  • Верхняя пластина из тонкой стали кладётся сверху, когда заряд уже накоплен трением. За счёт влияния на верхней поверхности образуется избыток отрицательного заряда, и он должен быть снят на заземлитель, чтобы при расстыковке двух обкладок не произошло взаимной компенсации.

Принцип действия этого плоского конденсатора должен быть уже понятен. Оператор трёт резину шерстью, оставляя на ней отрицательный заряд. Затем сверху кладётся кусок металла. Из-за значительной шероховатости поверхностей они не соприкасаются, но находятся на некотором расстоянии друг от друга. В результате металл электризуется влиянием. Электроны отталкиваются поверхностным зарядом резины и уходят на внешнюю плоскость, где оператор их снимает через заземлитель лёгким кратковременным прикосновением.

Низ металлической обкладки остаётся заряженным положительно. При расстыковке двух поверхностей этот эффект сохраняется, потому что в материале наблюдается дефицит электронов. И можно наблюдать искру, если дотронуться до металлической обкладки. Этот опыт можно на одном и том же заряде резины проделывать сотни раз, поскольку её поверхностное статическое сопротивление весьма велико. Это не даёт заряду растекаться. Демонстрируя этот опыт, Вольта привлёк внимание всего научного мира, но исследования никак не двигались вперёд, если не считать открытий Шарля Кулона.

В 1800 году сам Алессандро даёт толчок развитию изысканий в области электричества, изобретя свой знаменитый гальванический источник питания.

Конструкция плоского конденсатора

Электрофорус по сути представляет собой первый из когда-либо сконструированных плоских конденсаторов. Его обкладки способны хранить только статический заряд, потому что иначе наэлектризовать резину невозможно. Поверхность очень долго хранит электроны. Вольта даже предлагал снимать их пламенем свечи через ионизированный воздух или ультрафиолетовым излучением Солнца. Сегодня каждый школьник знает, что то же самое можно проделать и водой. Правда, электрофорус нужно будет после этого высушить.

В современном мире нижней обкладкой может служить тефлоновое покрытие или пластик. Они тоже хорошо набирают статический заряд. Диэлектриком здесь служит воздух. Чтобы перейти к конструкции современного конденсатора, нужно обе обкладки сделать металлическими. Тогда при возникновении на одной из них заряда влиянием электризация распространится на вторую, и если другой контакт заземлён, накопленная энергия может храниться какое-то время.


Запас электронов напрямую зависит от материала диэлектриков. Так например, среди современных конденсаторов встречаются:

  1. Слюдяные.
  2. Воздушные.
  3. Электролитические (оксидные).
  4. Керамические.

В эти названия как раз и заложен материал диэлектрика. От его состава зависит напрямую ёмкость, которая может быть увеличена во много раз. Роль диэлектриков объяснялась выше, в частности их параметры определяются непосредственно строением вещества. Однако многие материалы, обладающие высокими характеристиками, использовать не удаётся по причине их непригодности. Так например, вода обладает высокой диэлектрической проницаемостью.

КОНДЕНСАТОР — означает накопитель. В радио и электронной аппаратуре конденсатор является накопителем электрических зарядов. Простейший конденсатор состоит из двух металлических пластинок разделенных слоем диэлектрика. Диэлектрик — это материал который не проводит электрического тока и обладает определенными свойствами о которых поговорим чуть позже.

Так как конденсатор является накопителем, то он должен обладать определенной емкостью (объемом для накопления зарядов). На емкость конденсатора влияют площадь пластин (еще их называют «обкладками»), расстояние между обкладками и качество диэлектрика. К хорошим диэлектрикам относятся вакуум, эбонит, фарфор, слюда, полиэтилен, текстолит и много других синтетических материалов.
На рисунке изображен простейший конденсатор с двумя параллельными обкладками площадью S (S = m * n), которые находятся в вакууме на расстоянии d друг от друга.


Если между верхней и нижней обкладками конденсатора приложить напряжение Uab, то на верхней и нижней обкладках конденсатора накопятся одинаковые положительный +q и отрицательный -q заряды, которые называют свободными. Между обкладками возникает электрическое поле обозначенное на рисунке буквой Е.
Емкость нашего конденсатора (обозначается буквой С) будет: С = Eo*S/d, где Ео — электрическая постоянная (для вакуума) Ео=8,854 * 10 -12 Ф/м (Фарад на метр).
Если между обкладками поместить диэлектрик,


то ёмкость конденсатора будет: С = Er * Eo *S / d. В формуле расчета ёмкости добавилась величина Er — относительная диэлектрическая проницаемость введённого диэлектрика.
Из формулы следует, что емкость конденсатора увеличивается на величину Er проницаемости диэлектрика. Итак, чем больше площадь S пластин конденсатора, больше значение Er и меньше расстояние d между пластинами, тем больше емкость конденсатора. Основной единицей емкости в системе единиц СИ является фарад (Ф). Емкость 1Ф очень велика. В электротехнике обычно используют дольные единицы емкости:
микрофарада (мкФ), 1мкФ = 1*10 -6 Ф,
нанофарада (нФ), 1нФ = 1*10 -9 Ф, и
пикофарада (пФ), 1пФ = 1*10 -12 Ф.




При выборе диэлектрика для конденсаторов, кроме относительной диэлектрической проницаемости диэлектрика, учитывают еще два важных параметра:
1) Электрическую прочность — прочность диэлектрика при подаче на прокладки конденсатора высокого напряжения. При низкой электрической прочности может произойти электрический пробой, и диэлектрик станет проводником электрического тока;
2) Удельное объемное сопротивление — электрическое сопротивление диэлектрика постоянному току. Чем больше удельное сопротивление диэлектрика, тем меньше утечка накопленных зарядов в конденсаторе.

КОНДЕНСАТОР В ЦЕПИ ПОСТОЯННОГО ТОКА. На графике накопление заряда конденсатором выглядит как показано на рисунке 1.


Время заряда конденсатора зависит от ёмкости конденсатора (при одинаковом приложенном напряжении). Чем больше ёмкость конденсатора, тем больше время заряда. Аналогичная картина (Рис. 2) наблюдается при разрядке конденсатора на сопротивление. При одинаковом сопротивлении время разряда больше у конденсатора с большей ёмкостью.

КОНДЕНСАТОР В ЦЕПИ ПЕРЕМЕННОГО ТОКА. Если напряжение приложенное к емкостному элементу, будет изменяться по амплитуде (переменное напряжение),то будет изменяться и заряд конденсатора, то есть в емкостном элементе появится ток.



Ток Ic проходящий через конденсатор зависит от частоты f приложенного переменного напряжения и ёмкости С конденсатора. Если для постоянного тока сопротивление конденсатора можно считать равным бесконечности, то для переменного тока конденсатор обладает определённым сопротивлением. Сопротивление конденсатора переменному току Rc рассчитывается по формуле показанной на рисунке.
В формуле расчета емкостного сопротивления переменному току частота выражается в герцах, а емкость конденсатора в фарадах. Из формулы видно, что с увеличением частоты f при неизменной емкости конденсатора сопротивление Rc снижается, аналогично с увеличением емкости конденсатора при неизменной частоте сопротивление Rc так же снижается. Конденсаторы, так же как и резисторы, для получения заданной емкости Со можно включать параллельно и последовательно. Формулы расчета результирующей емкости показаны на рисунке.



КОНСТРУКЦИЯ, ПАРАМЕТРЫ И ТИПЫ КОНДЕНСАТОРОВ. Предположим, что мы конструируем конденсатор и попробуем, уже обладая определенными знаниями, рассчитать емкость конденсатора. Как известно, емкость конденсатора зависит от площади обкладок S, расстояния между обкладками d и диэлектрической проницаемости применяемого диэлектрика Er. Обкладки конденсатора изготавливаются из металлов с хорошей электрической проводимостью — алюминий, медь, серебро, золото. Емкость конденсатора не зависит от толщины обкладок, поэтому чем тоньше обкладки конденсатора, тем лучше — экономим металл и уменьшаем геометрический объём конденсатора.


Расстояние d не должно быть слишком малым, во избежание электрического пробоя диэлектрика.
Выберем в качестве диэлектрика наиболее распространенный материал — гетинакс с Er равной 6 … 8. Примем Er для нашего конденсатора равной 7.


Площадь S вычисляется для одной обкладки конденсатора при условии, что линейные размеры обкладок одинаковы. Если одна из обкладок имеет меньшие длину или ширину то площадь вычисляется для меньшей обкладки.
Все размеры — длина и ширина обкладок и расстояние между ними должны быть выражены в метрах. Примем размеры такие, какие показаны на рисунке. Подставим в формулу расчета емкости конденсатора наши данные: C = Er * Eo * S / d;
C = 7 * 8.854*10 -12 * 0.0025 / 0.001= 0.000000000155Ф (фарады).
Возведем полученный результат в 12 степень чтобы получить значение емкости в пикофарадах:
C = 0.000000000155 12 = 155пФ.
Полученная нами ёмкость конденсатора 155пф очень мала, обычно такие ёмкости используются в аппаратуре работающей на высоких частотах переменного тока порядка 1 — 600 МГц (мегагерц).
Представьте себе, что мы разрабатываем миниатюрный карманный радиоприемник в котором требуется порядка 30 таких конденсаторов.

Если мы установим в схему 30 разработанных нами конденсаторов, не считая других необходимых радиодеталей, то наш радиоприемник никак не получится миниатюрным. Все дело в том, что объём только наших конденсаторов получится таким, что его никак нельзя будет назвать приемлемым.
Объем одного конденсатора Vc равен Vc = 5см * 5см * 0,1см
Vc = 2,5см в кубе. Тогда объем 30 конденсаторов будет равен:
V = 30 * 2,5 = 75см в кубе.
Что делать, как быть, как уменьшить геометрический объем конденсатора для применения в миниатюрной радиоаппаратуре? Для решения этой проблемы максимально уменьшают расстояние между обкладками, тогда увеличивается емкость и уменьшается геометрический объем конденсатора. Но расстояние уменьшают до определенных пределов иначе конденсатор будет пробиваться даже при низком напряжении подаваемом на конденсатор. В связи с этим на каждом конденсаторе указывается напряжение которое он может выдержать.

Для уменьшения площади обкладок конденсатор делают многослойным состоящим как бы из нескольких параллельно включенных конденсаторов (вспомните формулу параллельного включения конденсаторов).
В качестве диэлектрика в миниатюрных конденсаторах используют тонкие пленки из синтетических материалов, а в качестве обкладок металлическую фольгу, чаще всего из алюминия.



На корпусе конденсатора, обычно, указывается его тип, емкость и рабочее напряжение. Остальные параметры конденсатора определяются из справочников. Емкость конденсатора указывается не так, как на электрических схемах. Например емкость 2,2пФ обозначается 2П2, емкость 1500 пФ — 1Н5, емкость 0,1 мкФ — М1, емкость 2,2 мкФ — 2М2, емкость 10 мкФ — 10М.
У обычных конденсаторов КМ, КД, МБМ и так далее трудно получить большую ёмкость при малых габаритах поэтому были разработаны так называемые электролитические конденсаторы у которых в качестве диэлектрика используется специальная электролитическая жидкость с очень большим Er. Ёмкость таких конденсаторов может достигать сотен тысяч микрофарад. К недостатку таких конденсаторов следует отнести низкое рабочее напряжение (до 500V) и обязательное соблюдение полярности при включении в схему.
Для настройки и подстройки некоторых типов радиоаппаратуры, например радиоприемник или телевизор, применяют специальные конденсаторы с изменяемой ёмкостью.

В зависимости от назначения такие конденсаторы называют «подстроечные» и «конденсаторы переменной емкости».
Емкость переменных и подстроечных конденсаторов изменяется механическим способом, путем изменения расстояния между обкладками или изменения площади пластин. В качестве диэлектрика в таких конденсаторах используется воздух или фарфор.
В заключение следует отметить, что в настоящее время, в связи с бурным развитием радиоэлектроники подстроечные и переменные конденсаторы практически не применяются. Их с успехом заменяют специальные фильтры и полупроводниковые приборы которые не требуют механического изменения параметров.

как найти, отчего зависит напряжение на этом элементе

Конденсатор — это электротехнический элемент, позволяющий накапливать заряд. Самая простая его форма представляет две пластины, разделенные слоем диэлектрика. Если на пластины подать напряжение, то оно сохранится какое-то время после его снятия. Важно знать, в чем измеряется емкость конденсатора, для правильного построения схем с этими элементами.

Применение в технике

Конденсаторы применяются в различной электро- и радиоаппаратуре. Эти элементы способны накапливать заряд и поддерживать напряжение (например, сетевое) на должном уровне во время незначительных перебоев с питанием. Конденсаторы большой емкости сами используются как питающие элементы для малогабаритной мобильной аппаратуры. Они еще называются ионисторы. Их недостатком является необходимость частого подзаряда.

Большое значение имеют эти элементы и в фильтрующих устройствах, приборах, задача которых не пропустить помехи в полезный сигнал, или уловить нужный сигнал в постоянном напряжении повышенного уровня.

Без конденсаторов не обходится ни один генератор переменного сигнала. Их назначение — задать частоту генерации, период и другие временные параметры. Здесь используются очень точные элементы, с допуском по номиналу не более 1%.

Конденсаторы бывают как постоянной, так и переменной емкости. Элементы переменной емкости используются в аппаратуре, требующей настройки на разные частоты. Например, это широко используется в настройке радиочастот в FM -приемниках.

Формулы для расчета конденсаторов

Для решения задач техники и прикладных теоретических расчетов нужно знать законы, по которым электрические величины взаимодействуют друг с другом. Эти законы выражаются формулами. Например, напряжение на конденсаторе зависит от его емкости и заряда, накопленного им.

Определение емкости

Это значение зависит от нескольких параметров. Чтобы его рассчитать, нужно знать, в чем измеряется емкость конденсатора. Эта величина эквивалентна тому, сколько кулон заряда накапливается элементом при напряжении в 1 вольт, приложенном к нему. Измеряется она в фарадах. Емкость этих элементов зависит также и от их формы.

  • Плоские конденсаторы — самая простая разновидность накопителей заряда. Как найти емкость конденсатора, имеющего плоскую форму, можно узнать, если определить все параметры, влияющие на это. На его емкость влияет расстояние между его обкладками (токопроводящие пластины) d, площадь самих обкладок S, диэлектрическая проницаемость вещества между обкладками ε и электрическая постоянная ε0, которая равна 8,85 ⋅ 10-12 фарад на метр. Формула конденсатора такова:

С = ε ⋅ ε0 ⋅ S/d

  • Цилиндрический конденсатор также состоит из двух заряженных обкладок, обе они имеют форму цилиндров, расположенных один внутри другого. Внутренний цилиндр цельный, внешний — полый. Расстояние между обкладками равно разности радиусов этих цилиндров. Формулу емкости конденсатора можно представить такой же, как в предыдущем случае, с той разницей, что площадь обкладок рассчитывается исходя из их высоты и радиуса:

С = 2 ⋅ π ⋅ ε ⋅ ε0 ⋅ h ⋅ R вн /(R нар — R вн) = ε ⋅ ε0 ⋅ S / d

где h — высота обкладки,

Rвн — внутренний радиус, R нар — наружный радиус,

π = 3,14.

  • Зарядом может обладать не только тело с двумя обкладками, но и проводящий шарообразный объект. Если подать на него напряжение, а потом измерить потенциал между ним и землей, то потенциал будет ненулевым. Формула для расчета шарообразного накопителя заряда:

С = 4 ⋅ π ⋅ ε ⋅ ε0 ⋅ R

где R — радиус шара.

Если в формулу подставить радиус Земли и диэлектрическую проницаемость воздуха, можно получить значение емкости Земли в фарадах. После расчетов:

С (Земли) = 700 микрофарад

Такую емкость могут иметь современные электролитические конденсаторы.

Если разместить один шар внутри другого и подать между ними напряжение, то полученная конструкция тоже будет накапливать заряд между поверхностями шаров. Определение емкости такой конструкции можно провести по формуле:

С = ε ⋅ ε0 ⋅4⋅π ⋅ R1 ⋅ R2 / (R2 — R1)

где R2 и R1 — радиусы соответствующих шарообразных поверхностей.

Емкость конденсатора зависит также и от типа используемого диэлектрика. Наиболее распространены керамические, электролитические, бумажные, воздушные и слюдяные наполнители.

Вычисление энергии

Накопители заряда обладают и другими параметрами. Один из них — это энергия. При зарядке конденсатора на его обкладках накапливается потенциальная энергия.

Она создаёт силу, притягивающую разноименно заряженные пластины, а также ток, который питает электроприборы, если использовать ионистор как источник питания. Энергию можно выразить как зависимость от напряжения обкладок и емкости:

W = C ⋅ U 2 /2

Ток утечки через диэлектрик

Ток утечки появляется в элементе, если есть пути протекания электрического тока с одной обкладки на другую. Чем менее изолирующими свойствами обладает диэлектрик, тем больше будет ток утечки. Особенно это применимо к конденсаторам с диэлектриком в виде промасленной бумаги. Этот параметр зависит и от конструкции элемента, и от загрязненности его корпуса. Если элемент негерметичен, ток утечки может увеличиваться при проникании влаги внутрь корпуса. Этот ток можно рассчитать по закону Ома:

I ут = U/R d

где I ут — ток утечки,

U — напряжение на обкладках,

R d — сопротивление изоляции диэлектрика.

Соединение элементов

При создании схем применяется различное соединение элементов. Элементы схемы могут быть соединены:

  • Параллельно;
  • Последовательно;
  • Параллельно — последовательно (смешанно).

Как найти ёмкость параллельно соединенных элементов? Нужно понять, что является общим при таком типе соединения. Так как напряжение прикладывается одновременно ко всем обкладкам, то оно является общим. Заряд же будет для каждого своим. По формуле:

q = C ⋅ U, здесь q — суммарный заряд, то есть

q = ΣC i ⋅ U = U ⋅ ΣC i

С общее будет равняться сумме всех С.

При последовательном соединении элементов общим для всех них будет заряд. В то же время напряжение будет для каждого из них разным, и общее будет складываться из всех по отдельности.

U = q / C, здесь U — сумма напряжений на всех элементах

U общее = q ⋅ Σ (1/ C i)

1/С общее = 1/С 1 +1/С 2 +… +1/C i

При таком соединении значение общей емкости будет меньше самого маленького значения этой величины в группе.

В случае использования смешанного соединения необходимо вычислить отдельно общую емкость для параллельного и отдельно для последовательного соединения. После этого по формуле последовательного соединения найти общее для двух получившихся величин значение.

Формула емкости конденсатора через напряжение.

Зарядка конденсатора от источника постоянной эдс

По назначению конденсатор можно сравнить с батарейкой. Но имеется принципиальное отличие в работе данных элементов. Существуют отличия в предельной емкости и скорости зарядки конденсатора и батарейки.

Формула заряда конденсатора

где q – величина заряда одной из обкладок конденсатора, а – разность потенциалов между его обкладками.

Электроемкость конденсатора — это величина, которая зависит то размеров и устройства конденсатора.

Заряд на пластинах плоского конденсатора равен:

где – электрическая постоянная; – площадь каждой (или наименьшей) пластины; – расстояние между пластинами; – диэлектрическая проницаемость диэлектрика, который находится между пластинами конденсатора.

Заряд на обкладках цилиндрического конденсатора вычисляется при помощи формулы:

где l – высота цилиндров; – радиус внешней обкладки; – радиус внутренней обкладки.

Заряд на обкладках сферического конденсатора найдем как:

Заряд конденсатора связан с энергией поля (W) внутри него:

Из формулы (6) следует, что заряд можно выразить как:

Рассмотрим последовательное соединение из N конденсаторов (рис. 1).

Здесь (рис.1) положительная обкладка одного конденсатора соединяется с отрицательной обкладкой следующего конденсатора. При таком соединении, обкладки соседних конденсаторов создают единый проводник. У всех конденсаторов, соединенных последовательно на обкладках имеются равные по величине заряды.

При параллельном соединении конденсаторов (рис.2), соединяют обкладки, имеющие заряды одного знака. Суммарный заряд соединения (q) равен сумме зарядов конденсаторов.

Примеры решения задач по теме «Заряд конденсатора»

ru.solverbook.com

Формула емкости конденсатора, С

Если q – величина заряда одной из обкладок конденсатора, а – разность потенциалов между его обкладками, то величина C, равная:

называется емкостью конденсатора. Это постоянная величина, которая зависит то размеров и устройства конденсатора.

Рассмотрим два одинаковых конденсатора, разница между которым заключается только в том, что между обкладками одного вакуум (или часто говорят воздух), между обкладками другого находится диэлектрик. В таком случае при равных зарядах на конденсаторах разность потенциалов воздушного конденсатора будет в раз меньше, чем между обкладками второго. Значит емкость конденсатора с диэлектриком (C) в раз больше, чем воздушного ():

где – диэлектрическая проницаемость диэлектрика.

За единицу емкости конденсатора принимают емкость такого конденсатора, который единичным зарядом (1 Кл) заряжается до разности потенциалов, равной одному вольту (в СИ). Единицей емкости конденсатора (как и любой эклектической емкости) в международной системе единиц (СИ) служит фарад (Ф).

Формула электрической емкости плоского конденсатора

Поле между обкладками плоского конденсатора обычно считают однородным. Его однородность нарушается только около краев. При вычислении емкости плоского конденсатора этими краевыми эффектами часто пренебрегают. Это следует делать, если расстояние между пластинами мало в сравнении с их линейными размерами. Для расчета емкости плоского конденсатора применяют формулу:

Электрическая емкость плоского конденсатора, который содержит N слоев диэлектрика толщина каждого , соответствующая диэлектрическая проницаемость i-го слоя , равна:

Формула электрической емкости цилиндрического конденсатора

Цилиндрический конденсатор представляется собой две соосных (коаксиальных) цилиндрические проводящие поверхности, разного радиуса, пространство между которыми заполняет диэлектрик. Электрическая емкость цилиндрического конденсатора вычисляется как:

Формула электрической емкости сферического конденсатора

Сферическим конденсатором называют конденсатор, обкладками которого являются две концентрические сферические проводящие поверхности, пространство между ними заполнено диэлектриком. Емкость такого конденсатора находят как:

где – радиусы обкладок конденсатора.

Примеры решения задач по теме «Емкость конденсатора»

ru.solverbook.com

Ёмкость конденсатора — Все формулы

Электрическая ёмкость — характеристика проводника (конденсатора), мера его способности накапливать электрический заряд.

Конденсатор состоит из двух проводников (обкладок), которые разделены диэлектриком. На емкость конденсатора не должны влиять окружающие тела, поэтому проводникам придают такую форму, чтобы поле, которое создается накапливаемыми зарядами, было сосредоточено в узком зазоре между обкладками конденсатора. Этому условию удовлетворяют: 1) две плоские пластины; 2) две концентрические сферы; 3) два коаксиальных цилиндра. Поэтому в зависимости от формы обкладок конденсаторы делятся на плоские, сферические и цилиндрические.

Так как поле сосредоточено внутри конденсатора, то линии напряженности начинаются на одной обкладке и кончаются на другой, поэтому свободные заряды, которые возникают на разных обкладках, равны по модулю и противоположны по знаку. Под емкостью конденсатора понимается физическая величина, равная отношению заряда Q, накопленного в конденсаторе, к разности потенциалов (φ1 — φ2) между его обкладками

Для получения больших ёмкостей конденсаторы соединяют параллельно. При этом напряжение между обкладками всех конденсаторов одинаково. Общая ёмкость батареи параллельно соединённых конденсаторов равна сумме ёмкостей всех конденсаторов, входящих в батарею.

Конденсаторы можно классифицировать по следующим признакам и свойствам:

1) по назначению — конденсаторы постоянной и переменной емкости;

2) по форме обкладок различают конденсаторы плоские, сферические, цилиндрические и др. ;

3) по типу диэлектрика — воздушные, бумажные, слюдяные, керамические, электролитические и т.д.

Так же есть:

Энергия конденсатора:

Ёмкость цилиндрического конденсатора:

Ёмкость плоского конденсатора:

Емкость сферического конденсатора:

В формуле мы использовали:

Электрическая ёмкость (ёмкость конденсатора)

Потенциал проводника (Напряжение)

Потенциал

Относительная диэлектрическая проницаемость

Электрическая постоянная

Площадь одной обкладки

Расстояние между обкладками

xn--b1agsdjmeuf9e.xn--p1ai

Заряд конденсатора, теория и примеры задач

Определение и заряд конденсатора

Возможность конденсатора накопить электрический заряд зависит от главной характеристики конденсатора – емкости (C).

По своему назначению конденсатор можно уподобить батарейке. Однако существует принципиальное отличие в принципах работы этих элементов. Отличаются, также максимальные емкости и скорости зарядки и разряда конденсатора и батарейки.

Если к конденсатору присоединить источник напряжения (рис.1), то на одной из пластин конденсатора станут накапливаться отрицательные заряды (электроны), на другой положительные частицы (положительные ионы). Между обкладками конденсатора находится диэлектрик, вследствие этого, заряды не могут перебраться на противоположную пластину. Однако заметим, что электроны двигаются от источника тока до пластины конденсатора.

При первоначальном соединении конденсатора и источника тока на обкладках конденсатора много свободного места. Это означает, что сопротивление току этот момент времени минимально, сам ток максимален. В ходе зарядки конденсатора сила тока в цепи постепенно падает, до того момента пока не закончится свободное место на обкладках. При полной зарядке конденсатора ток в цепи прекратится.

Время, которое затрачивается на зарядку конденсатора от нулевого заряда (максимального тока) до полностью заряженного конденсатора (минимальная или нулевая сила тока) называют переходным периодом заряда конденсатора. На практике процесс зарядки конденсатора считают законченным, если сила тока уменьшилась до 1% от начальной величины.

Величина заряда конденсатора (q) связана с его емкостью (C) и разностью потенциалов (U) между его обкладками как:

Примеры решения задач


ru.solverbook.com

Формула электроемкости конденсатора

Обкладки должны иметь такую форму и быть расположены так относительно друг друга, что поле, которое создается данной системой, было максимально сосредоточено в ограниченной области пространства, между обкладками.

Назначение конденсатора в том, чтобы накапливать и отдавать в электрической цепи заряд.

Основной характеристикой конденсатора является электрическая емкость (C). Электрическая емкость конденсатора – это взаимная емкость принадлежащих ему обкладок:

q – величина заряда на обкладке; – разность потенциалов между обкладками.

Электрическая ёмкость конденсатора зависит от диэлектрической проницаемости диэлектрика, который заполняет пространство между его обкладками. Если пространство между обкладками одного конденсатора заполнено диэлектриком с проницаемостью равной , а у второго конденсатора воздух между пластинами, то емкость конденсатора с диэлектриком (C) в раз больше, чем емкость воздушного конденсатора ():

Формула электроемкости основных типов конденсаторов

При расчете электроемкости плоского конденсатора нарушением однородности поля около краёв обкладок обычно пренебрегают. Это становится возможным, если расстояние между пластинами существенно меньше, чем линейные размеры обкладок. В таком случае электрическую емкость плоского конденсатора вычисляют при помощи формулы:

где – электрическая постоянная; S – площадь каждой (или наименьшей) пластины; d – расстояние между пластинами.

Если плоский конденсатор между обкладками имеет N слоев диэлектрика, при этом толщина каждого слоя равна , а диэлектрическая проницаемость , то его электрическую емкость рассчитывают при помощи формулы:

Цилиндрический конденсатор составляют две соосных (коаксиальных) цилиндрические проводящие поверхности, разного радиуса, пространство между которыми заполнено диэлектриком. При этом емкость цилиндрического конденсатора находят как:

где l – высота цилиндров; – радиус внешней обкладки; – радиус внутренней обкладки.

У сферического конденсатора обкладками служат две концентрические сферические проводящие поверхности, пространство обкладками заполняет диэлектрик. Емкость сферического конденсатора вычисляют как:

где – радиусы обкладок конденсатора. Если , то можно считать, что , тогда, мы имеем:

так как – площадь поверхности сферы, и если обозначить , то получим формулу для емкости плоского конденсатора (3). Если расстояние между обкладками сферического и цилиндрического конденсаторов малы (в сравнении с их радиусами), то в приближенных расчетах используют формулу емкости для плоского конденсатора.

Электрическую емкость для линии из двух проводов находят как:

где d – расстояние между осями проводов; R – радиус проводов; l – длина линии.

Формулы для вычисления электрической емкости соединений конденсаторов

Если конденсаторы соединены параллельно, то суммарная емкость батареи (C) находится как сумма емкостей отдельных конденсаторов ():

При последовательном соединении конденсаторов емкость батареи вычисляют как:

Если последовательно соединены N конденсаторов, с емкостями то емкость батареи найдем как:

Сопротивление конденсатора

Если конденсатор включен в цепь с постоянного тока, то сопротивление конденсатора можно считать бесконечно большим.

При включении конденсатора в цепь переменного тока, его сопротивление носит название емкостного, и вычисляют его с помощью формулы:

где – частота переменного тока; – угловая частота тока; C – емкость конденсатора.

Энергия поля конденсатора

Электрическое поле локализованное между пластинами конденсатора обладает энергией, которую можно вычислить при помощи формулы:

где –энергия поля конденсатора; q – заряд конденсатора; C – емкость конденсатора; – разность потенциалов между обкладками конденсатора.

Энергия поля плоского конденсатора:

Примеры решения задач по теме «Электроемкость конденсатора»

ru.solverbook.com

Как найти заряд конденсатора 🚩 как определить величину заряда 🚩 Естественные науки

В обычном (без плагинов и модов) варианте Minecraft такого понятия, как конденсатор, не существует. Вернее, устройство, выполняющее его функции, имеется, но название у него совершенно другое — компаратор. Некоторая путаница в этом плане произошла еще в период разработки такого прибора. Сперва — в ноябре 2012-го — представители Mojang (компании-создателя игры) объявили о скором появлении в геймплее конденсатора. Однако через месяц они высказались уже о том, что как такового этого прибора не будет, а вместо него в игре будет компаратор.

Подобное устройство существует для проверки заполненности расположенных позади него контейнеров. Таковыми могут быть сундуки (в том числе в виде ловушек), варочные стойки, раздатчики, выбрасыватели, печи, загрузочные воронки и т.п.

Помимо этого, его часто используют для сравнения двух сигналов редстоуна между собою — он выдает результат в соответствии с тем, как было запрограммировано в данной цепи, и с тем, какой режим выбран для самого механизма. В частности, компаратор может разрешить зажигание факела, если первый сигнал больше либо равен другому.

Также порой конденсатор-компаратор устанавливают рядом с проигрывателем, подключая его входом к последнему. Когда в звуковоспроизводящем устройстве проигрывается какая-либо пластинка, вышеупомянутый прибор будет выдавать сигнал, равный по силе порядковому номеру диска.

Скрафтить такой компаратор несложно, если имеется достаточно трудно добываемый ресурс — адский кварц. Его надо поставить в центральный слот верстака, над ним и по бокам от него установить три красных факела, а в нижнем ряду — такое же количество каменных блоков.

В большом количестве модов попадаются конденсаторы, имеющие самое разное предназначение. К примеру, в Galacticraft, где у геймеров есть возможность слетать на многие планеты для ознакомления с тамошними реалиями, появляется рецепт крафта кислородного конденсатора. Он служит для создания механизмов вроде коллектора и накопителя газа для дыхания, а также рамки воздушного шлюза. Для его изготовления четыре стальных пластины размещаются по углам верстака, в центре — оловянная канистра, а под нею — воздуховод. Остальные три ячейки занимают пластины из олова.

В JurassiCraft существует конденсатор потока — некий телепорт, позволяющий переместиться в удивительный игровой мир, кишащий динозаврами. Для создания такого прибора нужно поместить в два крайних вертикальных ряда шесть железных слитков, а в средний — два алмаза и между ними единицу пыли редстоуна. Дабы устройство заработало, надо поставить его на свинью либо вагонетку, а затем щелкнуть по нему правой клавишей мыши, быстро запрыгнув туда. При этом требуется поддержание высокой скорости устройства.

С модом Industrial Craft2 у игрока появляется возможность создавать как минимум два вида тепловых конденсаторов — красный и лазуритовый. Они служат исключительно для охлаждения ядерного реактора и для накопления его энергии и хороши для циклических сооружений такого типа. Остужаются они сами, соответственно, красной пылью или лазуритом.

Красный теплоконденсатор делается из семи единиц пыли редстоуна — их надо установить в виде буквы П и расставить под ними теплоотвод и теплообменник. Крафтинг же лазуритового устройства чуть посложнее. Для его создания четыре единицы пыли редстоуна расставляются по углам станка, в центр пойдет блок лазурита, по бокам от него — два красных тепловых конденсатора, сверху — теплоотвод реактора, а снизу — его же теплообменник.

В ThaumCraft, где сделан акцент на настоящем чародействе, конденсаторы тоже используются. Например, один из них — кристаллический — существует для аккумуляции и отдачи магии. Причем, что интересно, создавать его и многие другие вещи разрешено лишь после изучения особого элемента геймплея — исследования, проводимого за специальным столом и с определенными приборами.

Делается такой конденсатор из восьми тусклых осколков, в центр которых на верстаке помещается мистический деревянный блок. К сожалению, подобный прибор — равно как и его составляющие — просуществовал лишь до ThaumCraft 3, а в четвертой версии мода был упразднен.

www.kakprosto.ru

Соединение конденсаторов: формулы

Содержание:
  1. Последовательное соединение
  2. Онлайн калькулятор
  3. Смешанное соединение
  4. Параллельное соединение
  5. Видео

В электронных и радиотехнических схемах широкое распространение получило параллельное и последовательное соединение конденсаторов. В первом случае соединение осуществляется без каких-либо общих узлов, а во втором варианте все элементы объединяются в два узла и не связаны с другими узлами, если это заранее не предусмотрено схемой.

Последовательное соединение

При последовательном соединении два и более конденсаторов соединяются в общую цепь таким образом, что каждый предыдущий конденсатор соединяется с последующим лишь в одной общей точке. Ток (i), осуществляющий зарядку последовательной цепи конденсаторов будет иметь одинаковое значение для каждого элемента, поскольку он проходит только по единственно возможному пути. Это положение подтверждается формулой: i = ic1 = ic2 = ic3 = ic4.

В связи с одинаковым значением тока, протекающего через конденсаторы с последовательным соединением, величина заряда, накопленного каждым из них, будет одинаковой, независимо от емкости. Такое становится возможным, поскольку заряд, приходящий с обкладки предыдущего конденсатора, накапливается на обкладке последующего элемента цепи. Поэтому величина заряда у последовательно соединенных конденсаторов будет выглядеть следующим образом: Qобщ= Q1 = Q2 = Q3.

Если рассмотреть три конденсатора С1, С2 и С3, соединенные в последовательную цепь, то выясняется, что средний конденсатор С2 при постоянном токе оказывается электрически изолированным от общей цепи. В конечном итоге величина эффективной площади обкладок будет уменьшена до площади обкладок конденсатора с самыми минимальными размерами. Полное заполнение обкладок электрическим зарядом, делает невозможным дальнейшее прохождение по нему тока. В результате, движение тока прекращается во всей цепи, соответственно прекращается и зарядка всех остальных конденсаторов.

Общее расстояние между обкладками при последовательном соединении представляет собой сумму расстояний между обкладками каждого элемента. В результате соединения в последовательную цепь, формируется единый большой конденсатор, площадь обкладок которого соответствует обкладкам элемента с минимальной емкостью. Расстояние между обкладками оказывается равным сумме всех расстояний, имеющихся в цепи.

Падение напряжения на каждый конденсатор будет разным, в зависимости от емкости. Данное положение определяется формулой: С = Q/V, в которой емкость обратно пропорциональна напряжению. Таким образом, с уменьшением емкости конденсатора на него падает более высокое напряжение. Суммарная емкость всех конденсаторов вычисляется по формуле: 1/Cобщ = 1/C1 + 1/C2 + 1/C3.

Главная особенность такой схемы заключается в прохождении электрической энергии только в одном направлении. Поэтому в каждом конденсаторе значение тока будет одинаковым. Каждый накопитель в последовательной цепи накапливает равное количество энергии, независимо от емкости. То есть емкость может воспроизводиться за счет энергии, присутствующей в соседнем накопителе.

Онлайн калькулятор, для расчета емкости конденсаторов соединенных последовательно в электрической цепи.

Смешанное соединение

Параллельное соединение конденсаторов

Параллельным считается такое соединение, при котором конденсаторы соединяются между собой двумя контактами. Таким образом в одной точке может соединяться сразу несколько элементов.

Данный вид соединения позволяет сформировать единый конденсатор с большими размерами, площадь обкладок которого будет равна сумме площадей обкладок каждого, отдельно взятого конденсатора. В связи с тем, что емкость конденсаторов находится в прямой пропорциональной зависимости с площадью обкладок, общая емкость составить суммарное количество всех емкостей конденсаторов, соединенных параллельно. То есть, Собщ = С1 + С2 + С3.

Поскольку разность потенциалов возникает лишь в двух точках, то на все конденсаторы, соединенные параллельно, будет падать одинаковое напряжение. Сила тока в каждом из них будет отличаться, в зависимости от емкости и значения напряжения. Таким образом, последовательное и параллельное соединение, применяемое в различных схемах, позволяет выполнять регулировку различных параметров на тех или иных участках. За счет этого получаются необходимые результаты работы всей системы в целом.

electric-220.ru

Во всех электронных устройствах используются конденсаторы. При их конструировании или изготовлении своими руками параметры устройств рассчитываются по специальным формулам.

Расчёт конденсаторов

Один из главных параметров таких устройств – ёмкость. Рассчитать её можно по следующей формуле:

  • C – ёмкость,
  • q – заряд одной из обкладок элемента,
  • U – разность потенциалов между обкладками.

В электротехнике вместо понятия «разность потенциалов между обкладками» используется «напряжение на конденсаторе».

Ёмкость элемента не зависит от конструкции и размеров устройства, а только от напряжения на нём и заряда обкладок. Но эти параметры могут изменяться в зависимости от расстояния между ними и материала диэлектрика. Это учитывается в формуле:

С=Co*ε, где:

  • С – реальная ёмкость,
  • Со – идеальная, при условии, что между пластинами вакуум или воздух,
  • ε – диэлектрическая проницаемость материала между ними.

Например, если в качестве диэлектрика используется слюда, «ε» которой 6, то ёмкость такого устройства в 6 раз больше, чем воздушного, а при изменении количества диэлектрика меняются параметры конструкции. На этом принципе основана работа ёмкостного датчика положения.

Единицей ёмкости в системе СИ является 1 фарад (F). Это большая величина, поэтому чаще применяются микрофарады (1000000mkF=1F) и пикофарады (1000000pF=1mkF).

Расчет плоской конструкции

  • ε – диэлектрическая проницаемость изолирующего материала,
  • d – расстояние между пластинами.

Расчет конструкции цилиндрической формы

Цилиндрический конденсатор – это две соосные трубки различного диаметра, вставленные друг в друга. Между ними находится диэлектрик. При радиусе цилиндров, близком друг к другу и намного большем, чем расстояние между ними, цилиндрической формой можно пренебречь и свести расчёт к формуле, аналогичной той, по которой рассчитывается плоский конденсатор.

Вычисляются параметры такого устройства по формуле:

C=(2π*l*R*ε)/d, где:

  • l – длина устройства,
  • R – радиус цилиндра,
  • ε – диэлектрическая проницаемость изолятора,
  • d – его толщина.

Расчёт сферической конструкции

Есть устройства, обкладки которых представляют собой два шара, вложенные друг в друга. Формула ёмкости такого прибора:

C=(4π*l*R1*R2*ε)/(R2-R1), где:

  • R1 – радиус внутренней сферы,
  • R2 – радиус внешней сферы,
  • ε – диэлектрическая проницаемость.

Ёмкость одиночного проводника

Кроме конденсаторов, способностью накапливать заряд обладают отдельные проводники. Одиночным проводником считается такой проводник, который бесконечно далёк от других проводников. Параметры заряженного элемента рассчитывается по формуле:

  • Q – заряд,
  • φ – потенциал проводника.

Объём заряда определяется размером и формой устройства, а также окружающей средой. Материал прибора значения не имеет.

Способы соединения элементов

Не всегда есть в наличии элементы с необходимыми параметрами. Приходится соединять их различными способами.

Параллельное соединение

Это такое соединение деталей, при котором к одной клемме или контакту присоединяются первые обкладки каждого конденсатора. При этом вторые обкладки присоединяются к другой клемме.

При таком соединении напряжение на контактах всех элементов будет одинаковым. Заряд каждого из них происходит независимо от остальных, поэтому общая ёмкость равна сумме всех величин. Её находят по формуле:

где C1-Cn – параметры деталей, участвующих в параллельном соединении.

Важно! Конденсаторы имеют предельное допустимое напряжение, превышение которого приведёт к выходу элемента из строя. При параллельном соединении устройств с различным допустимым напряжением этот параметр получившейся сборки равен элементу с наименьшим значением.

Последовательное соединение

Это такое соединение, при котором к клемме присоединяется только одна пластина первого элемента. Вторая пластина присоединяется к первой пластине второго элемента, вторая пластина второго – к первой пластине третьего и так далее. Ко второй клемме присоединяется только вторая обкладка последнего элемента.

При таком соединении заряд на обкладках конденсатора в каждом приборе будет равен остальным, однако напряжение на них будет разным: для зарядки устройств большей ёмкости тем же зарядом требуется меньшая разность потенциалов. Поэтому вся цепочка представляет собой одну конструкцию, разность потенциалов которой равна сумме напряжений на всех элементах, а заряд конденсатора равен сумме зарядов.

Последовательное соединение увеличивает допустимое напряжение и уменьшает общую ёмкость, которая меньше самого меньшего элемента.

Рассчитываются эти параметры следующим образом:

  • Допустимое напряжение:

Uобщ=U1+U2+U3+…Un, где U1-Un – напряжение на конденсаторе;

  • Общая ёмкость:

1/Собщ=1/С1+1/С2+1/С3+…1/Сn, где С1-Сn – параметры каждого устройства.

Интересно. Если в цепи только два элемента, то можно воспользоваться упрощённой формулой: Собщ=(С1*С2)/(С1+С2).

Смешанное соединение

Это такое соединение, в котором есть детали, соединённые последовательно, и есть соединённые параллельно. Параметры всей цепи рассчитывается в следующей последовательности:

  1. определяются группы элементов, соединённые параллельно;
  2. для каждой группы в отдельности рассчитывается эквивалентные значения;
  3. рядом с каждой группой параллельно соединённых деталей пишутся получившиеся величины;
  4. получившаяся схема эквивалентна последовательной схеме и рассчитывается по соответствующим формулам.

Знание формул, по которым можно найти емкость при изготовлении конденсаторов или их соединении необходимо при конструировании электронных схем.

Видео

Содержание:

Одним из важных элементов электрической цепи является конденсатор, формулы для которого позволяют рассчитать и подобрать наиболее подходящий вариант. Основная функция данного устройства заключается в накоплении определенного количества электроэнергии. Простейшая система включает в себя два электрода или обкладки, разделенные между собой диэлектриком.

В чем измеряется емкость конденсатора

Одной из важнейших характеристик конденсатора является его емкость. Данный параметр определяется количеством электроэнергии, накапливаемой этим прибором. Накопление происходит в виде электронов. Их количество, помещающееся в конденсаторе, определяет величину емкости конкретного устройства.

Для измерения емкости применяется единица — фарада. Емкость конденсатора в 1 фараду соответствует электрическому заряду в 1 кулон, а на обкладках разность потенциалов равна 1 вольту. Эта классическая формулировка не подходит для практических расчетов, поскольку в конденсаторе собираются не заряды, а электроны. Емкость любого конденсатора находится в прямой зависимости от объема электронов, способных накапливаться при нормальном рабочем режиме. Для обозначения емкости все равно используется фарада, а количественные параметры определяются по формуле: С = Q / U, где С означает емкость, Q — заряд в кулонах, а U является напряжением. Таким образом, просматривается взаимная связь заряда и напряжения, оказывающих влияние на способность конденсатора к накоплению и удержанию определенного количества электричества.

Для расчетов используется формула:
в которой ε 0 = 8,854187817 х 10 -12 ф/м представляет собой постоянную величину. Прочие величины: ε — является диэлектрической проницаемостью диэлектрика, находящегося между обкладками, S — означает площадь обкладки, а d — зазор между обкладками.

Формула энергии конденсатора

С емкостью самым тесным образом связана другая величина, известная как . После зарядки любого конденсатора, в нем образуется определенное количество энергии, которое в дальнейшем выделяется в процессе разрядки. С этой потенциальной энергией вступают во взаимодействие обкладки конденсатора. В них образуются разноименные заряды, притягивающиеся друг к другу.

В процессе зарядки происходит расходование энергии внешнего источника для разделения зарядов с положительным и отрицательным значением, которые, затем располагаются на обкладках конденсатора. Поэтому в соответствии с законом сохранения энергии, она не исчезает бесследно, а остается внутри конденсатора в виде электрического поля, сосредоточенного между пластинами. Разноименные заряды образуют взаимодействие и последующее притяжение обкладок между собой.

Каждая пластина конденсатора под действием заряда создает напряженность электрического поля, равную Е/2. Общее поле будет складываться из обоих полей, возникающих в каждой обкладке с одинаковыми зарядами, имеющими противоположные значения.

Таким образом, энергия конденсатора выражается формулой: W=q(E/2)d. В свою очередь, напряжение выражается с помощью понятий напряженности и расстояния и представляется в виде формулы U=Ed. Это значение, подставленное в первую формулу, отображает энергию конденсатора в таком виде:W=qU/2. Для получения окончательного результата необходимо использовать определение емкости: C=q/U, и в конце концов энергия заряженного конденсатора будет выглядеть следующим образом: W эл = CU 2 /2.

Формула заряда конденсатора

Для выполнения зарядки, конденсатор должен быть подключен к цепи постоянного тока. С этой целью может использоваться генератор. У каждого генератора имеется внутреннее сопротивление. При замыкании цепи происходит зарядка конденсатора. Между его обкладками появляется напряжение, равное электродвижущей силе генератора: U c = E.

Обкладка, подключенная к положительному полюсу генератора, заряжается положительно (+q), а другая обкладка получает равнозначный заряд с отрицательной величиной (- q). Величина заряда q находится в прямой пропорциональной зависимости с емкостью конденсатора С и напряжением на обкладках Uc. Эта зависимость выражается формулой: q = C x Uc.

В процессе зарядки одна из обкладок конденсатора приобретает, а другая теряет определенное количество электронов. Они переносятся по внешней цепи под влиянием электродвижущей силы генератора. Такое перемещение является электрическим током, известным еще как зарядный емкостной ток (Iзар).

Течение зарядного тока в цепи происходит практически за тысячные доли секунды, до того момента, пока напряжение конденсатора не станет равным электродвижущей силе генератора. Напряжение увеличивается плавно, а потом постепенно замедляется. Далее значение напряжения конденсатора будет постоянным. Во время зарядки по цепи течет зарядный ток. В самом начале он достигает максимальной величины, так как напряжение конденсатора имеет нулевое значение. Согласно закона Ома I зар = Е/R i , поскольку к сопротивлению Ri приложена вся ЭДС генератора.

Формула тока утечки конденсатора

Ток утечки конденсатора вполне можно сравнить с воздействием подключенного к нему резистора с каким-либо сопротивлением R. Ток утечки тесно связан с типом конденсатора и качеством используемого диэлектрика. Кроме того, важным фактором становится конструкция корпуса и степень его загрязненности.

Некоторые конденсаторы имеют негерметичный корпус, что приводит к проникновению влаги из воздуха и возрастанию тока утечки. В первую очередь это касается устройств, где в качестве диэлектрика использована промасленная бумага. Значительные токи утечки возникают из-за снижения электрического сопротивления изоляции. В результате нарушается основная функция конденсатора — способность получать и сохранять заряд электрического тока.

Основная формула для расчета выглядит следующим образом: I ут = U/R d , где I ут, — это ток утечки, U — напряжение, прилагаемое к конденсатору, а R d — сопротивление изоляции.

§ 6. Заряд и разряд конденсатора

Чтобы зарядить конденсатор, надо, чтобы свободные электроны перешли из одной обкладки на другую. Переход электронов с одной обкладки конденсатора на другую происходит под действием напряжения источника по проводам, соединяющим этот источник с обкладками конденсатора.

В момент включения конденсатора зарядов на его обкладках нет и напряжение на нем равно нулю μ с =0. Поэтому зарядный ток определяется внутренним сопротивлением источника r в и имеет наибольшую величину:

I З max =E/ r в.

По мере накопления зарядов на обкладках конденсатора напряжение на нем увеличивается и падение напряжения на внутреннем сопротивлении источника будет равно разности ЭДС источника и напряжения на конденсаторе (Е- μ с). следовательно, зарядный ток

i з =(Е- μ с)/ r в.

Таким образом, с увеличением напряжения на конденсаторе ток заряда снизится и при μ с =Е становится равным нулю. Процесс изменения напряжения на конденсаторе и тока заряда во времени изображен на рис. 1. В самом начале заряда напряжение на конденсаторе резко возрастает, так как зарядный ток имеет наибольшее значение и накопление зарядов на обкладках конденсатора происходит интенсивно. По мере повышения напряжения на конденсаторе зарядный ток уменьшается и накопление зарядов на обкладках замедляется. Продолжительность заряда конденсатора зависит от его емкости и сопротивления цепи, увеличение которых приводит к возрастанию продолжительности заряда. С увеличением емкости конденсатора, возрастает количество зарядов, накапливаемых на его пластинах, а если увеличить сопротивление цепи уменьшится и зарядный ток, а это замедляет процесс накопления зарядов на этих обкладках.

Если обкладки заряженного конденсатора подключить к какому-либо сопротивлению R , то за счет напряжения на конденсаторе будет протекать разрядный ток конденсатора. При разряде конденсатора электронысодной пластины (при их избытке) будут переходить на другую (при их недостатке) и будет продолжается до тех пор, пока потенциалы обкладок не выравняются, т. е. напряжение на конденсаторе станет равным нулю. Изменение напряжения в процессе разряда конденсатора изображено на рис. 2. Ток разряда конденсатора пропорционален напряжению на конденсаторе (i р =μ с /R ), и его изменение во времени подобно изменению напряжения.



В начальный момент разряда напряжение на конденсаторе наибольшее (μ с =Е) и разрядный ток максимальный (I р max =E /R ), так что разряд происходит быстро. При понижении напряжения, ток разряда снижается и процесс перехода зарядов с одной обкладки на другую затормаживается.

Время процесса разряда конденсатора зависит от сопротивления цепи и емкости конденсатора, причем возрастание как сопротивления, так и емкости увеличивает продолжительность разряда. С увеличением сопротивления разрядный ток снижается, замедляется процесс переноски зарядов с одной на другую обкладок; с увеличением емкости конденсатора повышается заряд на обкладках.

Таким образом, в цепи, содержащей конденсатор, ток проходит только в процессе его заряда и разряда, т. е. когда напряжение на обкладках претерпевает изменение во времени. При постоянстве напряжения ток через конденсатор не проходит, т. е. конденсатор не пропускает постоянный ток, так как между его обкладками помещен диэлектрик и в результате этого цепь разомкнута.

При зарядке конденсатора, последний способен накапливать электрическую энергию, потребляя ее от энергоисточника. Накопленная энергия сохраняется определенное время. При разряде конденсатора эта энергия переходит к разрядному резистору, нагревая его, т. е. энергию электрического поля превращается в тепловую. Чем выше емкость конденсатора и напряжение на его обкладках, тем будет больше энергии, запасенной на нем. Энергия электрического поля конденсатора определяется следующим выражением

W=CU 2 /2.

Если конденсатор емкостью 100 мкФ заряжен до напряжения 200 В, то энергия, запасенная в электрическом поле конденсатора, W =100· 10 -6 · 200 2 /2=2 Дж.

Вам понадобится

  • — знание емкости или геометрических и физических параметров конденсатора;
  • — знание энергии или заряда на конденсаторе.

Инструкция

Найдите напряжение между пластинами конденсатора, если известна текущая величина накопленной им энергии, а также его емкость. Энергия, запасенная конденсатором, может быть вычислена по формуле W=(C∙U²)/2, где C — емкость, а U — напряжение между пластинами. Таким образом, значение напряжения может быть получено как корень из удвоенного значения энергии, деленного на емкость. То есть, оно будет равно: U=√(2∙W/C).

Энергия, запасенная конденсатором, также может быть вычислена на основании значения содержащегося в нем заряда (количества ) и напряжения между обкладками. Формула, задающая соответствие между этими параметрами, имеет вид: W=q∙U/2 (где q — заряд). Следовательно, зная энергию и , можно вычислить напряжение между его пластинами по формуле: U=2∙W/q.

Поскольку заряд на конденсаторе пропорционален как приложенному к его пластинам напряжению, так и емкости устройства (он определяется формулой q=C∙U), то, зная заряд и емкость, можно найти и напряжение. Соответственно, для проведения расчета используйте формулу: U=q/C.

Для получения значения напряжения на конденсаторе с известными геометрическими и параметрами, сначала рассчитайте его емкость. Для простого плоского конденсатора, состоящего из двух проводящих пластин, разделенных , расстояние между которыми пренебрежимо мало по сравнению с их размерами, емкость может быть вычислена по формуле: C=(ε∙ε0∙S)/d.-12 Ф/м), ε — относительная диэлектрическая проницаемость пространства между пластинами (ее можно узнать из физических справочников). Вычислив емкость, рассчитайте напряжение одним из методов, приведенных в шагах 1-3.

Обратите внимание

Для получения корректных результатов при вычислении напряжений между обкладками конденсаторов, перед проведением расчетов приводите значения всех параметров в систему СИ.

Для того чтобы знать, можно ли использовать в том или ином месте схемы конденсатор, следует определить его . Способ нахождения этого параметра зависит от того, каким образом он обозначен на конденсаторе и обозначен ли вообще.

Вам понадобится

  • Измеритель емкости

Инструкция

На крупных конденсаторах емкость обычно обозначена открытым текстом: 0,25 мкФ или 15 uF. В этом случае, способ ее определения тривиален.

На менее крупных конденсаторах (в том , SMD) емкость двумя или тремя цифрами. В первом случае, она обозначена в пикофарадах. Во втором случае, первые две цифры емкость , а третья — в каких единицах она выражена:1 — десятки пикофарад;
2 — сотни пикофарад;
3 — нанофарады;
4 — десятки нанофарад;
5 — доли микрофарады.

Существует также система обозначения емкости, использующая сочетания латинских букв и цифр. Буквы обозначают следующие цифры:A — 10;
B — 11;
C — 12;
D — 13;
E — 15;
F — 16;
G — 18;
H — 20;
J — 22;
K — 24;
L — 27;
M — 30;
N — 33;
P — 36;
Q — 39;
R — 43;
S — 47;
T — 51;
U — 56;
V — 62;
W — 68;
X — 75;
Y — 82;
Z — 91.Полученное число следует умножить на число 10, предварительно возведенное в степень, равную цифре, следующей после . Результат будет выражен в пикофарадах.

Встречаются конденсаторы, емкость на которых не обозначена вообще. Вы наверняка встречали их, в , в стартерах ламп дневного . В этом случае, измерить емкость можно только специальным прибором. Они цифровыми и мостовыми.В любом случае, если конденсатор впаян в то или иное устройство, его следует обесточить, разрядить в нем конденсаторы фильтра и сам конденсатор, емкость которого следует измерить, и лишь после этого выпаять его. Затем его необходимо подключить к прибору.На цифровом измерителе сначала выбирают самый грубый предел, затем переключают его до тех пор, пока он не покажет перегрузку. После этого переключатель переводят на один предел назад и читают показания, а по положению переключателя определяют, в каких единицах они выражены.На мостовом измерителе, последовательно переключая , на каждом из них прокручивают регулятор из одного конца шкалы в другой, пока звук из динамика не исчезнет. Добившись исчезновения , по шкале регулятора считывают результат, а единицы, в которых он выражен, также определяют по положению переключателя.Затем конденсатор устанавливают обратно в устройство.

Обратите внимание

Никогда не подключайте к измерителю заряженные конденсаторы.

Источники:

  • Справочник по системам обозначения емкости

Найти значение электрического заряда можно двумя способами. Первый – измерить силу взаимодействия неизвестного заряда с известным и с помощью закона Кулона рассчитать его значение. Второй – внести заряд в известное электрическое поле и измерить силу, с которой оно действует на него. Для измерения заряда протекающего через поперечное сечение проводника за определенное время измерьте силу тока и умножьте ее на значение времени.

Вам понадобится

  • чувствительный динамометр, секундомер, амперметр, измеритель электростатического поля, воздушный конденсатор.

Инструкция

Измерение заряда при его с известным зарядомЕсли известен одного тела, поднесите к нему неизвестный заряд и измерьте между ними в метрах. Заряды начнут взаимодействовать. С помощью динамометра измерьте силу их взаимодействия. Рассчитайте значение неизвестного заряда — для этого квадрат измеренного расстояния умножьте на значение силы и поделите на известный заряд.9)). Если заряды отталкиваются, то они одноименные, если же притягиваются – разноименные.

Измерение значения заряда , внесенного в электрическое полеИзмерьте значение постоянного электрического поля специальным прибором (измеритель электрического поля). Если такого прибора нет, возьмите воздушный конденсатор, зарядите его, измерьте напряжение на его обкладках и поделите не расстояние между пластинами – это и будет значение электрического поля внутри конденсатора в вольтах на метр. Внесите в поле неизвестный заряд. С помощью чувствительного динамометра измерьте силу, которая на него действует. Измерение проводите в . Поделите значение силы на напряженность электрического поля. Результатом будет значение заряда в Кулонах (q=F/Е).

Измерение заряда , протекающего через поперечное проводникаСоберите электрическую цепь с проводниками и последовательно подключите к ней амперметр. Замкните ее на источник тока и измерьте силу тока с помощью амперметра в амперах. Одновременно секундомером засеките , в которого в цепи был электрический ток. Умножив значение силы тока на полученное время, узнайте заряд, через поперечное сечение каждого за это время (q=I t). При измерениях следите, чтобы проводники не перегревались и не произошло короткое замыкание.

Конденсатором называется устройство, способное накапливать электрические заряды. Количество накапливаемой электрической энергии в конденсаторе характеризуется его емкостью . Она измеряется в фарадах. Считается, что емкость в один фарад соответствует конденсатору, заряженному электрическим зарядом в один кулон при разности потенциалов на его обкладках в один вольт.

Инструкция

Определите емкость плоского конденсатора по формуле С = S e e0/d, где S — площадь поверхности одной пластины, d — между пластинами, e — относительная диэлектрическая проницаемость , заполняющей пространство между пластинами (в вакууме она равна ), e0 — электрическая постоянная, равная 8,854187817 10(-12) Ф/м.Исходя из приведенной формулы, величина емкости будет зависеть от площади проводников, между ними и от материала диэлектрика. В качестве диэлектрика может применяться или слюда.

Вычислите емкость сферического конденсатора по формуле С = (4П e0 R²)/d, где П — число «пи», R — радиус сферы, d — величина зазора между его сферами.Величина емкости сферического конденсатора прямо пропорциональна концентрической сферы и обратно пропорциональна расстоянию между сферами.

Рассчитайте емкость цилиндрического конденсатора по формуле С = (2П e e0 L R1)/(R2-R1), где L — длина конденсатора , П — число «пи», R1 и R2 — радиусы его цилиндрических обкладок.

Если конденсаторы в цепи соединены параллельно, рассчитайте их общую емкость по формуле С = С1+С2+…+Сn, где С1, С2,…Сn – емкости параллельно соединенных конденсаторов.

Вычислите общую емкость последовательно соединенных конденсаторов по формуле 1/С = 1/С1+1/С2+…+1/Сn, где С1, С2,…Сn — емкости последовательно соединенных конденсаторов.

Обратите внимание

На любом конденсаторе обязательно должна быть нанесена маркировка, которая может быть буквенно-цифровая или цветовая. Маркировка отражает его параметры.

Источники:

  • Цветовая маркировка резисторов, конденсаторов и индуктивностей

Емкость – величина, в системе СИ выражаемая в фарадах. Хотя используются, фактически, лишь производные от нее – микрофарады, пикофарады и так далее. Что касается электроемкости плоского конденсатора, она зависит от зазора меж обкладок и их площади, от вида диэлектрика, в данном зазоре расположенного.

Инструкция

В том случае, если обкладки конденсатора имеют одинаковую площадь и имеют расположение строго одна над другой, рассчитайте площадь одной из обкладок – любой. Если же одна из них относительно другой смещена либо они разные , нужно рассчитывать площадь области, в которой обкладки друг дружку перекрывают.

При этом используются общепринятые формулы, рассчитывать площади таких геометрических фигур, как круг (S=π(R^2)), прямоугольника (S=ab), его частного случая – квадрата (S=a^2) – и других.(-12) Ф/м и является, по сути, диэлектрической проницаемостью вакуума.

Калькулятор емкости последовательного соединения конденсаторов • Электротехнические и радиотехнические калькуляторы • Онлайн-конвертеры единиц измерения

Калькулятор позволяет рассчитать емкость нескольких конденсаторов, соединенных последовательно.

Пример. Рассчитать эквивалентную емкость двух соединенных последовательно конденсаторов 10 мкФ и 5 мкФ.

Входные данные

Добавить конденсатор

Выходные данные

Эквивалентная емкость

C микрофарад (мкФ)

Введите значения емкости в поля C1 и C 2, добавьте при необходимости новые поля, выберите единицы емкости (одинаковые для всех полей ввода) в фарадах (Ф), миллифарадах (мФ), микрофарадах (мкФ), пикофарадах (пФ), нанофарадах (нФ) и нажмите на кнопку Рассчитать.

1 мФ = 0,001 Ф. 1 мкФ = 0,000001 = 10⁻⁶ Ф. 1 нФ = 0,000000001 = 10⁻⁹ Ф. 1 пФ = 0,000000000001 = 10⁻¹² Ф.

В соответствии со вторым правилом Кирхгофа, падения напряжения V₁, V₂ and V₃ на каждом из конденсаторов в группе из трех соединенных последовательно конденсаторов в общем случае различные и общая разность потенциалов V равна их сумме:

По определению емкости и с учетом того, что заряд Q группы последовательно соединенных конденсаторов является общим для всех конденсаторов, эквивалентная емкость Ceq всех трех конденсаторов, соединенных последовательно, определяется как

или

Для группы из n соединенных последовательно конденсаторов эквивалентная емкость Ceq равна величине, обратной сумме величин, обратных емкостям отдельных конденсаторов:

или

Эта формула для Ceq и используется для расчетов в этом калькуляторе. Например, общая емкость соединенных последовательно трех конденсаторов емкостью 10, 15 and 20 мкФ будет равна 4,62 мкФ:

Если конденсаторов только два, то их общая емкость определяется по формуле

или

Если имеется n соединенных последовательно конденсаторов с емкостью C, их эквивалентная емкость равна

Отметим, что для расчета общей емкости нескольких соединенных последовательно конденсаторов используется та же формула, что и для расчета общего сопротивления параллельно соединенных резисторов.

Отметим также, что общая емкость группы из любого количества последовательно соединенных конденсаторов всегда будет меньше, чем емкость самого маленького конденсатора, а добавление конденсаторов в группу всегда приводит к уменьшению емкости.

Конденсаторы на печатной плате

Отдельного упоминания заслуживает падение напряжения на каждом конденсаторе в группе последовательно соединенных конденсаторов. Если все конденсаторы в группе имеют одинаковую номинальную емкость, падение напряжения на них скорее всего будет разным, так как конденсаторы в реальности будут иметь разную емкость и разный ток утечки. На конденсаторе с наименьшей емкостью будет наибольшее падение напряжения и, таким образом, он будет самым слабым звеном этой цепи.

Выравнивающие резисторы уменьшают разброс напряжений на отдельных конденсаторах

Для получения более равномерного распределения напряжений параллельно конденсаторам включают выравнивающие резисторы. Эти резисторы работают как делители напряжения, уменьшающие разброс напряжений на отдельных конденсаторах. Но даже с этими резисторами все равно для последовательного включения следует выбирать конденсаторы с большим запасом по рабочему напряжению.

Если несколько конденсаторов соединены параллельно, разность потенциалов V на группе конденсаторов равна разности потенциалов соединительных проводов группы. Общий заряд Q разделяется между конденсаторами и если их емкости различны, то заряды на отдельных конденсаторах Q₁, Q₂ and Q₃ тоже будут различными. Общий заряд определяется как

Конденсаторы, соединенные параллельно

По определению емкости, эквивалентная емкость группы конденсаторов равна

отсюда

или

Для группы n включенных параллельно конденсаторов

То есть, если несколько конденсаторов включены параллельно, их эквивалентная емкость определяется путем сложения емкостей всех конденсаторов в группе.

Возможно, вы заметили, что конденсаторы ведут себя противоположно резисторам: если резисторы соединены последовательно, их общее сопротивление всегда будет выше сопротивлений отдельных резисторов, а в случае конденсаторов всё происходит с точностью до наоборот.

Конденсаторы на печатной плате

Калькулятор энергии конденсатора

Это калькулятор энергии конденсатора, простой инструмент, который поможет вам оценить количество энергии, хранящейся в конденсаторе. Вы также можете узнать, сколько заряда накопилось в конденсаторе. Читайте дальше, чтобы узнать, какая энергия хранится в конденсаторе и каково уравнение энергии конденсатора.

Какая энергия хранится в конденсаторе?

Конденсатор — это электронный компонент, обычно используемый в схемах. Его функция — накапливать электрический заряд .В стандартных конденсаторах с параллельными пластинами на соседних пластинах присутствуют заряды равной, но противоположной величины (для сферических конденсаторов вместо пластин используются концентрические сферы). Эти заряды создают между собой электрическое поле, состоящее из определенного количества энергии контура. Поскольку мы говорим о накопленных зарядах, это пример потенциальной энергии.

Формула энергии конденсатора

Как вы оцениваете энергию E , запасенную в конденсаторе с емкостью C и приложенным напряжением В ? Это эквивалентно работе, выполняемой батареей по перемещению заряда Q на конденсатор.Получающееся уравнение:

E = 1/2 * C * V² .

Используя общую формулу для емкости, C = Q / V , мы можем переписать уравнение емкости для энергии в двух других аналогичных формах:

E = 1/2 * Q² / C или E = 1/2 * Q * V .

Электрическая энергия в конденсаторе — пример

Сколько энергии может храниться в конденсаторе емкостью C = 300 мкФ , когда мы подключаем его к источнику напряжения В = 20 В ? Давайте вместе разберемся!

  • Чтобы облегчить нашу жизнь, используйте научное обозначение емкости: C = 3 · 10⁻⁴ F
  • По формуле мощности емкости результат оценивается как: E = 1/2 * 3 · 10⁻⁴ F * (20 В) ² = 6 · 10⁻² Дж
  • Энергия, запасенная в конденсаторе, также может быть записана как 0.06 Дж или 60 мДж
  • Кроме того, мы можем оценить общий заряд, накопленный в конденсаторе: Q = C * V = 3 · 10⁻⁴ F * 20 V = 6 · 10⁻³ C = 6 мКл
  • … или вы можете просто сэкономить свое время, используя этот калькулятор энергии конденсатора, который автоматически выполняет все вычисления за вас!

Кстати, если у вас есть система с более чем одним конденсатором, вам лучше проверить наши конденсаторы последовательно или конденсаторы в параллельных калькуляторах, чтобы быстро найти общую емкость, потому что это значение, которое вы должны использовать в формуле для энергия конденсатора.

Преобразования энергии в LC-контуре

LC-цепь — это система, состоящая из катушки индуктивности и конденсатора. На практике это можно обобщить как цепь RLC из-за некоторого сопротивления в системе. Как только схема обрабатывает сигнал резонансной частоты, потенциальная энергия конденсатора непрерывно преобразуется в магнитную энергию, создаваемую током, протекающим через катушку. Эти виды схем широко используются при обработке сигналов или при отправке и приеме радиоволн.

Конденсаторы и диэлектрики | Физика II

Цели обучения

К концу этого раздела вы сможете:

  • Опишите действие конденсатора и определите емкость.
  • Объясните, почему конденсаторы с параллельными пластинами и их емкости.
  • Обсудите процесс увеличения емкости диэлектрика.
  • Определите емкость при заданном заряде и напряжении.

Конденсатор — это устройство, используемое для хранения электрического заряда.Конденсаторы имеют разные применения: от фильтрации статического электричества при радиоприеме до накопления энергии в дефибрилляторах сердца. Обычно в промышленных конденсаторах две токопроводящие части расположены близко друг к другу, но не соприкасаются, как показано на рисунке 1. (В большинстве случаев между двумя пластинами используется изолятор для обеспечения разделения — см. Обсуждение диэлектриков ниже). Клеммы батареи подключены к изначально незаряженному конденсатору, равные количества положительного и отрицательного заряда, + Q и — Q , разделены на его две пластины.Конденсатор в целом остается нейтральным, но в этом случае мы называем его хранящим заряд Q .

Рис. 1. Оба конденсатора, показанные здесь, были изначально разряжены перед подключением к батарее. Теперь у них разделены заряды + Q и — Q на двух половинах. (а) Конденсатор с параллельными пластинами. (b) Скрученный конденсатор с изоляционным материалом между двумя проводящими листами.

Конденсатор

Конденсатор — это устройство, используемое для хранения электрического заряда.

Количество заряда Q , которое может хранить конденсатор , зависит от двух основных факторов — приложенного напряжения и физических характеристик конденсатора, таких как его размер.

Количество заряда

Q Конденсатор может хранить

Количество заряда Q , которое может хранить конденсатор , зависит от двух основных факторов — приложенного напряжения и физических характеристик конденсатора, таких как его размер.

Рис. 2. Силовые линии электрического поля в этом конденсаторе с параллельными пластинами, как всегда, начинаются с положительных зарядов и заканчиваются отрицательными.Поскольку напряженность электрического поля пропорциональна плотности силовых линий, она также пропорциональна количеству заряда на конденсаторе.

Система, состоящая из двух идентичных параллельных проводящих пластин, разделенных расстоянием, как на рисунке 2, называется конденсатором с параллельными пластинами . Легко увидеть взаимосвязь между напряжением и накопленным зарядом для конденсатора с параллельными пластинами, как показано на рисунке 2. Каждая линия электрического поля начинается с отдельного положительного заряда и заканчивается отрицательным, так что поля будет больше. линии, если есть больше заряда.(Рисование одной силовой линии для каждого заряда — это только удобство. Мы можем нарисовать много силовых линий для каждого заряда, но их общее количество пропорционально количеству зарядов.) Таким образом, напряженность электрического поля прямо пропорциональна Ом. .

Поле пропорционально начислению:

E Q ,

, где символ ∝ означает «пропорционально». Из обсуждения в разделе «Электрический потенциал в однородном электрическом поле» мы знаем, что напряжение на параллельных пластинах равно

.

V = Ed .

Таким образом, V E . Отсюда следует, что V Q и, наоборот,

Q V .

В целом это верно: чем больше напряжение, приложенное к любому конденсатору, тем больше в нем хранится заряд.

Различные конденсаторы будут накапливать разное количество заряда для одного и того же приложенного напряжения в зависимости от их физических характеристик. Мы определяем их емкость C так, чтобы заряд Q , хранящийся в конденсаторе, был пропорционален C .Заряд, накопленный в конденсаторе, равен

.

Q = CV .

Это уравнение выражает два основных фактора, влияющих на количество накопленного заряда. Этими факторами являются физические характеристики конденсатора C и напряжение В . Изменив уравнение, мы видим, что емкость C — это величина заряда, накопленного на вольт, или

.

[латекс] C = \ frac {Q} {V} \\ [/ latex].

Емкость

Емкость C — это величина накопленного заряда на вольт, или

[латекс] C = \ frac {Q} {V} \\ [/ latex]

Единица измерения емкости — фарад (Ф), названная в честь Майкла Фарадея (1791–1867), английского ученого, внесшего вклад в области электромагнетизма и электрохимии. Поскольку емкость — это заряд на единицу напряжения, мы видим, что фарад — это кулон на вольт, или

.

[латекс] 1 \ text {F} = \ frac {1 \ text {C}} {1 \ text {V}} \\ [/ latex].

Конденсатор емкостью 1 фарад может хранить 1 кулон (очень большое количество заряда) при приложении всего 1 вольт. Таким образом, одна фарада — это очень большая емкость. Типичные конденсаторы варьируются от долей пикофарада (1 пФ = 10 −12 Ф) до миллифарадов (1 мФ = 10 −3 Ф).

На рисунке 3 показаны некоторые распространенные конденсаторы. Конденсаторы в основном изготавливаются из керамики, стекла или пластика, в зависимости от назначения и размера. Как обсуждается ниже, в их конструкции обычно используются изоляционные материалы, называемые диэлектриками.

Рисунок 3. Некоторые типичные конденсаторы. Размер и значение емкости не обязательно связаны. (Источник: Windell Oskay)

Параллельный пластинчатый конденсатор

Рис. 4. Конденсатор с параллельными пластинами, разделенные пластинами на расстояние d. Каждая пластина имеет площадь A.

Конденсатор с параллельными пластинами, показанный на рисунке 4, имеет две идентичные проводящие пластины, каждая из которых имеет площадь поверхности A , разделенных расстоянием d (без материала между пластинами).Когда на конденсатор подается напряжение В , он сохраняет заряд Q , как показано. Мы можем увидеть, как его емкость зависит от A и d , рассмотрев характеристики кулоновской силы. Мы знаем, что одинаковые заряды отталкиваются, в отличие от зарядов притягиваются, и сила между зарядами уменьшается с расстоянием. Поэтому кажется вполне разумным, что чем больше пластины, тем больше заряда они могут хранить, потому что заряды могут расходиться больше. Таким образом, C должен быть больше для большего A .Точно так же, чем ближе пластины расположены друг к другу, тем сильнее на них притяжение противоположных зарядов. Таким образом, C должно быть больше для меньшего d .

Можно показать, что для конденсатора с параллельными пластинами есть только два фактора ( A, и d ), которые влияют на его емкость C . Емкость конденсатора с параллельными пластинами в форме уравнения равна

.

[латекс] C = \ epsilon_ {o} \ frac {A} {d} \\ [/ latex].

Емкость параллельного пластинчатого конденсатора

[латекс] C = \ epsilon_ {o} \ frac {A} {d} \\ [/ latex]

A — это площадь одной пластины в квадратных метрах, а d — это расстояние между пластинами в метрах.Константа ε 0 — диэлектрическая проницаемость свободного пространства; его числовое значение в единицах СИ составляет ε 0 = 8,85 × 10 −12 Ф / м. Единицы измерения Ф / м эквивалентны C 2 / Н · м 2 . Небольшое числовое значение ε 0 связано с большим размером фарада. Конденсатор с параллельными пластинами должен иметь большую площадь, чтобы его емкость приближалась к фарадам. (Обратите внимание, что приведенное выше уравнение действительно, когда параллельные пластины разделены воздухом или свободным пространством.Когда между пластинами помещается другой материал, уравнение изменяется, как обсуждается ниже.)

Пример 1. Емкость и заряд в параллельном пластинчатом конденсаторе

  1. Какова емкость конденсатора с параллельными пластинами, каждая из которых имеет площадь 1,00 м 2 , разделенных расстоянием 1,00 мм?
  2. Какой заряд хранится в этом конденсаторе, если к нему приложено напряжение 3,00 × 10 3 В?
Стратегия

Определение емкости C представляет собой прямое приложение уравнения [латекс] C = \ epsilon_ {o} \ frac {A} {d} \\ [/ latex].{-9} \ text {F} = 8.85 \ text {nF} \ end {array} \\ [/ latex]

Обсуждение части 1

Это небольшое значение емкости указывает на то, насколько сложно сделать устройство с большой емкостью. Помогают специальные методы, например, использование тонких пленок очень большой площади, расположенных близко друг к другу.

Решение для Части 2

Заряд любого конденсатора определяется уравнением Q = CV . Ввод известных значений в это уравнение дает

[латекс] \ begin {array} {lll} Q & = & CV = \ left (8.{3} \ text {V} \ right) \\\ text {} & = & 26.6 \ mu \ text {C} \ end {array} \\ [/ latex]

Обсуждение части 2

Этот заряд лишь немного больше, чем у обычного статического электричества. Поскольку воздух разрывается при примерно 3,00 × 10 6 В / м, на этом конденсаторе не может быть накоплено больше заряда за счет увеличения напряжения.

Другой интересный биологический пример, связанный с электрическим потенциалом, обнаружен в плазматической мембране клетки. {6} \ text {V / m} \\ [/ latex]

Этого электрического поля достаточно, чтобы вызвать пробой в воздухе.

Диэлектрик

Предыдущий пример подчеркивает сложность сохранения большого количества заряда в конденсаторах. Если d сделать меньше, чтобы получить большую емкость, то максимальное напряжение должно быть уменьшено пропорционально, чтобы избежать пробоя (поскольку [латекс] E = \ frac {V} {d} \\ [/ latex]). Важным решением этой проблемы является размещение изоляционного материала, называемого диэлектриком , между пластинами конденсатора и обеспечение минимально возможного размера d .Мало того, что меньший d увеличивает емкость, многие изоляторы могут выдерживать более сильные электрические поля, чем воздух, перед тем, как сломаться.

Есть еще одно преимущество использования диэлектрика в конденсаторе. В зависимости от используемого материала емкость больше, чем заданная уравнением [латекс] C = \ kappa \ epsilon_ {0} \ frac {A} {d} \\ [/ latex], на коэффициент κ , называемый диэлектрическая проницаемость . Конденсатор с параллельными пластинами с диэлектриком между пластинами имеет емкость, определяемую выражением [латекс] C = \ kappa \ epsilon_ {0} \ frac {A} {d} \\ [/ latex] (конденсатор с параллельными пластинами с диэлектриком).

Значения диэлектрической проницаемости κ для различных материалов приведены в таблице 1. Обратите внимание, что κ для вакуума равно 1, поэтому приведенное выше уравнение справедливо и в этом случае. Если использовать диэлектрик, например, поместив тефлон между пластинами конденсатора в примере 1, то емкость будет больше в κ раз, что для тефлона составляет 2,1.

Эксперимент на вынос: создание конденсатора

Насколько большой конденсатор можно сделать из обертки от жевательной резинки? Пластины будут из алюминиевой фольги, а разделитель (диэлектрик) между ними — из бумаги.

Таблица 1. Диэлектрическая проницаемость и диэлектрическая прочность для различных материалов при 20ºC
Материал Диэлектрическая проницаемость κ Диэлектрическая прочность (В / м)
Вакуум 1,00000
Воздух 1.00059 3 × 10 6
Бакелит 4,9 24 × 10 6
Плавленый кварц 3.78 8 × 10 6
Неопреновый каучук 6,7 12 × 10 6
Нейлон 3,4 14 × 10 6
Бумага 3,7 16 × 10 6
Полистирол 2,56 24 × 10 6
Стекло Pyrex 5,6 14 × 10 6
Кремниевое масло 2.5 15 × 10 6
Титанат стронция 233 8 × 10 6
Тефлон 2,1 60 × 10 6
Вода 80

Обратите внимание, что диэлектрическая проницаемость воздуха очень близка к 1, так что конденсаторы, заполненные воздухом, действуют так же, как конденсаторы с вакуумом между пластинами за исключением , что воздух может стать проводящим, если напряженность электрического поля слишком большой.(Напомним, что [латекс] E = \ frac {V} {d} \\ [/ latex] для конденсатора с параллельными пластинами.) В таблице 1 также показаны максимальные напряженности электрического поля в В / м, которые называются диэлектрической прочностью , для нескольких материалов. Это поля, над которыми материал начинает разрушаться и проводить. Диэлектрическая прочность накладывает ограничение на напряжение, которое может быть приложено для данного расстояния между пластинами. 6 \ text {V / m} \ right) \ left ( 1.{-3} \ text {m} \ right) \\\ text {} & = & 3000 \ text {V} \ end {array} \\ [/ latex]

Однако предел для расстояния 1,00 мм, заполненного тефлоном, составляет 60 000 В, поскольку диэлектрическая прочность тефлона составляет 60 × 10 6 В / м. Таким образом, тот же конденсатор, заполненный тефлоном, имеет большую емкость и может подвергаться гораздо большему напряжению. Используя емкость, которую мы рассчитали в приведенном выше примере для конденсатора с параллельными пластинами, заполненного воздухом, мы обнаружили, что конденсатор с тефлоновым заполнением может хранить максимальный заряд

[латекс] \ begin {array} {lll} Q & = & CV \\\ text {} & = & \ kappa {C} _ {\ text {air}} V \\\ text {} & = & (2.4 \ text {V}) \\\ text {} & = & 1.1 \ text {mC} \ end {array} \\ [/ latex]

Это в 42 раза больше заряда того же конденсатора, заполненного воздухом.

Диэлектрическая прочность

Максимальная напряженность электрического поля, при превышении которой изолирующий материал начинает разрушаться и становится проводником, называется его диэлектрической прочностью.

Микроскопически, как диэлектрик увеличивает емкость? За это отвечает поляризация изолятора. Чем легче он поляризуется, тем больше его диэлектрическая проницаемость κ .Вода, например, представляет собой полярную молекулу , потому что один конец молекулы имеет небольшой положительный заряд, а другой конец имеет небольшой отрицательный заряд. Полярность воды обуславливает ее относительно большую диэлектрическую проницаемость, равную 80. Эффект поляризации лучше всего объясняется характеристиками кулоновской силы. На рис. 5 схематично показано разделение зарядов в молекулах диэлектрического материала, помещенных между заряженными пластинами конденсатора. Кулоновская сила между ближайшими концами молекул и зарядом на пластинах притягивает и очень сильна, поскольку они расположены очень близко друг к другу.Это притягивает больше заряда к пластинам, чем если бы пространство было пустым, а противоположные заряды находились на расстоянии d .

Рис. 5. (a) Молекулы изоляционного материала между пластинами конденсатора поляризованы заряженными пластинами. Это создает слой противоположного заряда на поверхности диэлектрика, который притягивает больше заряда к пластине, увеличивая ее емкость. (б) Диэлектрик снижает напряженность электрического поля внутри конденсатора, что приводит к уменьшению напряжения между пластинами при одинаковом заряде.Конденсатор сохраняет тот же заряд при меньшем напряжении, что означает, что он имеет большую емкость из-за диэлектрика.

Другой способ понять, как диэлектрик увеличивает емкость, — это рассмотреть его влияние на электрическое поле внутри конденсатора. На рисунке 5 (b) показаны силовые линии электрического поля с установленным диэлектриком. Поскольку силовые линии заканчиваются зарядами в диэлектрике, их меньше, идущих от одной стороны конденсатора к другой. Таким образом, напряженность электрического поля меньше, чем если бы между пластинами был вакуум, даже если бы на пластинах был одинаковый заряд.Напряжение между пластинами составляет В, = Ед, , поэтому оно тоже снижается за счет диэлектрика. Таким образом есть меньшее напряжение В, для того же заряда Q ; поскольку [латекс] C = \ frac {Q} {V} \\ [/ latex], емкость C больше.

Диэлектрическая проницаемость обычно определяется как [латекс] \ kappa = \ frac {E_0} {E} \\ [/ latex], или отношение электрического поля в вакууме к электрическому полю в диэлектрическом материале, и в конечном итоге связанных с поляризуемостью материала.

Великие и малые вещи: субмикроскопическое происхождение поляризации

Поляризация — это разделение зарядов в атоме или молекуле. Как уже отмечалось, планетарная модель атома описывает его как имеющее положительное ядро, вращающееся вокруг отрицательных электронов, аналогично планетам, вращающимся вокруг Солнца. Хотя эта модель не совсем точна, она очень полезна для объяснения широкого круга явлений и будет уточнена в других местах, например, в атомной физике. Субмикроскопическое происхождение поляризации можно смоделировать, как показано на рисунке 6.

Рис. 6. Художественное представление о поляризованном атоме. Орбиты электронов вокруг ядра слегка смещены внешними зарядами (показаны в преувеличении). Получающееся разделение зарядов внутри атома означает, что он поляризован. Обратите внимание, что непохожий заряд теперь ближе к внешним зарядам, вызывая поляризацию.

В атомной физике мы обнаружим, что орбиты электронов более правильно рассматривать как электронные облака с плотностью облака, связанной с вероятностью обнаружения электрона в этом месте (в отличие от определенных местоположений и путей движения планет на их орбитах). вокруг Солнца).Это облако сдвигается кулоновской силой, так что в среднем атом имеет разделенный заряд. Хотя атом остается нейтральным, теперь он может быть источником кулоновской силы, поскольку заряд, поднесенный к атому, будет ближе к одному типу заряда, чем к другому.

Некоторым молекулам, например молекулам воды, присуще разделение зарядов, поэтому они называются полярными молекулами. На рисунке 7 показано разделение зарядов в молекуле воды, которая имеет два атома водорода и один атом кислорода (H 2 O).Молекула воды несимметрична — атомы водорода отталкиваются в одну сторону, придавая молекуле форму бумеранга. Электроны в молекуле воды более сконцентрированы вокруг более заряженного ядра кислорода, чем вокруг ядер водорода. Это делает кислородный конец молекулы слегка отрицательным, а водородный конец — слегка положительным. Внутреннее разделение зарядов в полярных молекулах облегчает их выравнивание с внешними полями и зарядами. Следовательно, полярные молекулы обладают более сильными поляризационными эффектами и имеют более высокие диэлектрические проницаемости.Те, кто изучает химию, обнаружат, что полярная природа воды имеет множество эффектов. Например, молекулы воды собирают ионы гораздо эффективнее, потому что у них есть электрическое поле и разделение зарядов, чтобы притягивать заряды обоих знаков. Кроме того, как было показано в предыдущей главе, полярная вода обеспечивает защиту или экранирование электрических полей в сильно заряженных молекулах, представляющих интерес в биологических системах.

Рис. 7. Художественная концепция молекулы воды. Существует внутреннее разделение зарядов, поэтому вода — полярная молекула.Электроны в молекуле притягиваются к ядру кислорода и оставляют избыток положительного заряда около двух ядер водорода. (Обратите внимание, что схема справа является приблизительной иллюстрацией распределения электронов в молекуле воды. На ней не показано фактическое количество протонов и электронов, участвующих в структуре.)

Исследования PhET: лаборатория конденсаторов

Узнайте, как работает конденсатор! Измените размер пластин и добавьте диэлектрик, чтобы увидеть влияние на емкость.Измените напряжение и посмотрите, как на пластинах накапливаются заряды. Наблюдайте за электрическим полем в конденсаторе. Измерьте напряжение и электрическое поле.

Щелкните, чтобы загрузить симуляцию. Запускать на Java.

Сводка раздела

  • Конденсатор — это устройство, используемое для хранения заряда.
  • Количество заряда Q , которое может хранить конденсатор, зависит от двух основных факторов — приложенного напряжения и физических характеристик конденсатора, таких как его размер.
  • Емкость C — это количество накопленного заряда на вольт, или [латекс] C = \ frac {Q} {V} \\ [/ latex].
  • Емкость конденсатора с параллельными пластинами составляет [латекс] C = {\ epsilon} _ {0} \ frac {A} {d} \\ [/ latex], когда пластины разделены воздухом или свободным пространством. [latex] {\ epsilon} _ {\ text {0}} [/ latex] называется диэлектрической проницаемостью свободного пространства.
  • Конденсатор с параллельными пластинами с диэлектриком между пластинами имеет емкость, определяемую выражением [латекс] C = \ kappa \ epsilon_ {0} \ frac {A} {d} \\ [/ latex], где κ — диэлектрик. константа материала.
  • Максимальная напряженность электрического поля, при превышении которой изолирующий материал начинает разрушаться и становится проводником, называется диэлектрической прочностью.

Концептуальные вопросы

  1. Зависит ли емкость устройства от приложенного напряжения? А как насчет хранящегося в нем заряда?
  2. Используйте характеристики кулоновской силы, чтобы объяснить, почему емкость должна быть пропорциональна площади пластины конденсатора. Аналогичным образом объясните, почему емкость должна быть обратно пропорциональна расстоянию между пластинами.
  3. Объясните причину, по которой диэлектрический материал увеличивает емкость по сравнению с тем, что было бы с воздухом между пластинами конденсатора.Какова независимая причина того, что диэлектрический материал также позволяет приложить большее напряжение к конденсатору? (Таким образом, диэлектрик увеличивает C и допускает более V .)
  4. Как полярный характер молекул воды помогает объяснить относительно большую диэлектрическую проницаемость воды? (См. Рисунок 7.)
  5. Искры возникают между пластинами заполненного воздухом конденсатора при более низком напряжении, когда воздух влажный, чем когда сухой. Объясните почему, учитывая полярный характер молекул воды.
  6. Вода имеет большую диэлектрическую проницаемость, но редко используется в конденсаторах. Объяснить, почему.
  7. Мембраны в живых клетках, в том числе в человеческих, характеризуются разделением заряда через мембрану. Таким образом, мембраны представляют собой заряженные конденсаторы, важные функции которых связаны с разностью потенциалов на мембране. Требуется ли энергия для разделения этих зарядов в живых мембранах, и если да, то является ли ее источником метаболизм пищевой энергии или каким-либо другим источником?

Рисунок 8.Полупроницаемая мембрана клетки имеет разную концентрацию ионов внутри и снаружи. Диффузия перемещает ионы K + (калий) и Cl (хлорид) в показанных направлениях, пока кулоновская сила не остановит дальнейший перенос. Это приводит к слою положительного заряда снаружи, слою отрицательного заряда внутри и, следовательно, к напряжению на клеточной мембране. Мембрана обычно непроницаема для Na + (ионы натрия).

Задачи и упражнения

  1. Какой заряд сохраняется в конденсаторе 180 мкФ, когда к нему приложено 120 В?
  2. Найдите накопленный заряд, когда 5.50 В подается на конденсатор емкостью 8,00 пФ.
  3. Какой заряд хранится в конденсаторе в Примере 1?
  4. Рассчитайте напряжение, приложенное к конденсатору 2,00 мкФ, когда он имеет заряд 3,10 мкКл.
  5. Какое напряжение необходимо приложить к конденсатору 8,00 нФ для накопления заряда 0,160 мкКл?
  6. Какая емкость необходима для хранения 3,00 мкКл заряда при напряжении 120 В?
  7. Какая емкость терминала большого генератора Ван-де-Граафа, учитывая, что он хранит 8?00 мкКл заряда при напряжении 12,0 МВ?
  8. Найдите емкость конденсатора с параллельными пластинами, площадь пластин которого составляет 5,00 м 2 , разделенных слоем тефлона 0,100 мм.
  9. (a) Какова емкость конденсатора с параллельными пластинами, площадь пластин которого составляет 1,50 м 2 , разделенных 0,0200 мм неопренового каучука? (b) Какой заряд он держит, когда к нему приложено 9,00 В?
  10. Комплексные концепции. Шутник подает 450 В на 80.Конденсатор 0 мкФ, а затем бросает его ничего не подозревающей жертве. Палец пострадавшего обгорел от разряда конденсатора через 0,200 г мяса. Какое повышение температуры мяса? Разумно ли предполагать отсутствие изменения фазы?
  11. Необоснованные результаты. (a) Определенный конденсатор с параллельными пластинами имеет пластины площадью 4,00 м 2 , разделенные 0,0100 мм нейлона, и накапливает 0,170 Кл заряда. Какое приложенное напряжение? б) Что неразумного в этом результате? (c) Какие допущения являются ответственными или противоречивыми?

Глоссарий

конденсатор: устройство, накапливающее электрический заряд

емкость: количество заряда на единицу вольт

диэлектрик: изоляционный материал

диэлектрическая прочность: максимальное электрическое поле, выше которого изоляционный материал начинает разрушаться и проводить

конденсатор с параллельными пластинами: две идентичные проводящие пластины, разделенные расстоянием

полярная молекула: молекула с внутренним разделением заряда

Избранные решения проблем и упражнения

1.21,6 мК

3. 80.0 мС

5. 20,0 кВ

7. 667 пФ

9. (а) 4,4 мкФ; (б) 4.0 × 10 −5 C

11. (а) 14,2 кВ; (b) Напряжение неоправданно велико, более чем в 100 раз больше напряжения пробоя нейлона; (c) Предполагаемый заряд неоправданно велик и не может храниться в конденсаторе таких размеров.

энергии в конденсаторах | Физика

Цели обучения

К концу этого раздела вы сможете:

  • Перечислите некоторые варианты использования конденсаторов.
  • Выразите в виде уравнения энергию, запасенную в конденсаторе.
  • Объясните функцию дефибриллятора.

Большинство из нас видели инсценировки, в которых медицинский персонал использовал дефибриллятор , чтобы пропустить электрический ток через сердце пациента, чтобы заставить его нормально биться. (Просмотрите рис. 1.) Часто реалистичный в деталях, человек, применяющий электрошок, просит другого человека «сделать на этот раз 400 джоулей». Энергия, передаваемая дефибриллятором, накапливается в конденсаторе и может регулироваться в зависимости от ситуации.Часто используются единицы СИ — джоули. Менее драматично использование конденсаторов в микроэлектронике, например в некоторых портативных калькуляторах, для подачи энергии при зарядке аккумуляторов. (См. Рис. 1.) Конденсаторы также используются для питания ламп-вспышек на камерах.

Рис. 1. Энергия, накопленная в большом конденсаторе, используется для сохранения памяти электронного калькулятора, когда его батареи заряжены. (Источник: Kucharek, Wikimedia Commons)

Энергия, запасенная в конденсаторе, представляет собой электрическую потенциальную энергию, и, таким образом, она связана с зарядом Q и напряжением В на конденсаторе.Мы должны быть осторожны при применении уравнения для электрической потенциальной энергии ΔPE = q Δ V к конденсатору. Помните, что ΔPE — это потенциальная энергия заряда q , проходящего через напряжение Δ В . Но конденсатор начинает с нулевого напряжения и постепенно достигает своего полного напряжения по мере зарядки. Первый заряд, помещенный на конденсатор, испытывает изменение напряжения Δ В = 0, поскольку конденсатор имеет нулевое напряжение в незаряженном состоянии.Последний заряд, помещенный на конденсатор, испытывает Δ В = В , поскольку теперь на конденсаторе имеется полное напряжение В . Среднее напряжение на конденсаторе во время процесса зарядки составляет [латекс] \ frac {V} {2} \\ [/ latex], поэтому среднее напряжение, испытываемое при полной зарядке q , равно [latex] \ frac {V} {2} \\ [/ латекс]. Таким образом, энергия, запасенная в конденсаторе, E cap , равна [latex] E _ {\ text {cap}} = \ frac {QV} {2} \\ [/ latex], где Q — это заряд на конденсаторе приложено напряжение В и .2} {2C} \\ [/ latex],

, где Q — заряд, В, — напряжение, а C — емкость конденсатора. Энергия выражается в джоулях для заряда в кулонах, напряжения в вольтах и ​​емкости в фарадах.

В дефибрилляторе доставка большого заряда коротким импульсом к набору лопастей на груди человека может быть спасением. Инфаркт у человека мог возникнуть в результате быстрого, нерегулярного сердцебиения — фибрилляции сердца или желудочков.Применение сильного разряда электрической энергии может прекратить аритмию и позволить кардиостимулятору тела вернуться к нормальному режиму. Сегодня в машинах скорой помощи обычно есть дефибриллятор, который также использует электрокардиограмму для анализа сердечного ритма пациента. Автоматические внешние дефибрилляторы (AED) можно найти во многих общественных местах (рис. 2). Они предназначены для использования непрофессионалами. Устройство автоматически диагностирует состояние сердца пациента, а затем применяет разряд с соответствующей энергией и формой волны.Во многих случаях перед использованием АВД рекомендуется СЛР.

Рис. 2. Автоматические внешние дефибрилляторы можно найти во многих общественных местах. {2}} {2C} \\ [/ latex], где Q — это заряд, В, — напряжение, а С — емкость конденсатора.Энергия выражается в джоулях, когда заряд — в кулонах, напряжение — в вольтах, а емкость — в фарадах.

Концептуальные вопросы

  1. Как изменяется энергия, содержащаяся в заряженном конденсаторе, когда вставлен диэлектрик, если конденсатор изолирован и его заряд постоянен? Означает ли это, что работа была сделана?
  2. Что происходит с энергией, накопленной в конденсаторе, подключенном к батарее, когда вставлен диэлектрик? Была ли проделана работа в процессе?

Задачи и упражнения

  1. (a) Какая энергия хранится в 10.0 мкФ конденсатор дефибриллятора сердца заряжен до
    9,00 × 10 3 В? (b) Найдите количество сохраненного заряда.
  2. При операции на открытом сердце гораздо меньшее количество энергии вызывает дефибрилляцию сердца. (а) Какое напряжение приложено к конденсатору 8,00 мкФ дефибриллятора сердца, который накапливает 40,0 Дж энергии? (b) Найдите количество сохраненного заряда.
  3. Конденсатор емкостью 165 мкФ используется вместе с двигателем. Сколько энергии в нем хранится при подаче 119 В?
  4. Предположим, у вас есть 9.Батарея 00 В, конденсатор 2,00 мкФ и конденсатор 7,40 мкФ. (а) Найдите заряд и запасенную энергию, если конденсаторы подключены к батарее последовательно. (б) Сделайте то же самое для параллельного подключения.
  5. Нервный физик опасается, что две металлические полки его книжного шкафа с деревянным каркасом могут получить высокое напряжение, если они заряжены статическим электричеством, возможно, вызванным трением. (а) Какова емкость пустых полок, если они имеют площадь 1,00 × 10 2 м 2 и равны 0.200 м друг от друга? (б) Какое напряжение между ними, если на них помещены противоположные заряды величиной 2,00 нКл? (c) Чтобы показать, что это напряжение представляет небольшую опасность, рассчитайте запасенную энергию.
  6. Покажите, что для данного диэлектрического материала максимальная энергия, которую может хранить конденсатор с параллельными пластинами, прямо пропорциональна объему диэлектрика (Объем = A · d ). Обратите внимание, что приложенное напряжение ограничено диэлектрической прочностью.
  7. Создайте свою проблему. Рассмотрим дефибриллятор сердца, аналогичный описанному в примере 1. Постройте задачу, в которой вы исследуете заряд, накопленный в конденсаторе дефибриллятора, как функцию накопленной энергии. Среди факторов, которые необходимо учитывать, — это приложенное напряжение и то, должно ли оно меняться в зависимости от подаваемой энергии, диапазон задействованных энергий и емкость дефибриллятора. Вы также можете рассмотреть гораздо меньшую энергию, необходимую для дефибрилляции во время операции на открытом сердце, как вариант решения этой проблемы.
  8. Необоснованные результаты. (a) В определенный день для запуска двигателя грузовика требуется 9,60 × 10 3 Дж электроэнергии. Вычислите емкость конденсатора, способного хранить такое количество энергии при напряжении 12,0 В. (б) Что неразумного в этом результате? (c) Какие допущения ответственны?

Глоссарий

Дефибриллятор : устройство, используемое для поражения электрическим током сердца пострадавшего от сердечного приступа с целью восстановления нормального ритмического паттерна сердца.

Избранные решения проблем и упражнения

1.(а) 405 Дж; (б) 90,0 мС

2. (а) 3,16 кВ; (б) 25,3 мС

4. (а) 1.42 × 10 −5 C, 6.38 × 10 −5 Дж; (б) 8.46 × 10 −5 C, 3.81 × 10 −4 J

5. а — 4.43 × 10 –12 F; б) 452 В; (в) 4.52 × 10 –7 Дж

8. (а) 133 F; (б) Такой конденсатор был бы слишком большим для перевозки в грузовике. Размер конденсатора был бы огромным; (c) Неразумно предполагать, что конденсатор может хранить необходимое количество энергии.

Емкость и разделение пластин

Dynamics Track
Наклонная плоскость
Импульс

Конденсатор
Пластина Sep
Пластина Sep / Вольт
Диэлектрики

Цепи
Закон Ом
Последовательность / Параллель

Wave Tank
Частота / длина волны
Two Pt Interf.

Оптическая скамья
Рефракция
Фокусное расстояние

Емкость и разделение пластин

Конденсатор с параллельными пластинами

Конденсатор с параллельными пластинами — это устройство, используемое для изучения конденсаторов.Это сводит к минимуму функцию конденсатора. Конденсаторы в реальном мире обычно скручены по спирали в небольших корпусах, поэтому конденсатор с параллельными пластинами значительно упрощает привязку функции к устройству.

Конденсатор работает, накапливая противоположные заряды на параллельных пластинах, когда напряжение подается с одной пластины на другую. Между пластинами существует электрическое поле, которое позволяет конденсатору накапливать энергию. Количество заряда, которое может храниться на один приложенный вольт, определяется площадью поверхности пластин и расстоянием между ними.Чем больше пластины и чем ближе они расположены, тем больше заряда может храниться на каждый вольт разности потенциалов между пластинами.

Количество заряда, которое может храниться в конденсаторе, измеряется его емкостью. Конденсатор в один фарад (F) может хранить один кулон заряда на каждый вольт, приложенный к конденсатору. Формула для этого:

C = q / v

Где C — емкость в фарадах, q — заряд в кулонах, а v — электрический потенциал в вольтах.

Для конденсатора с параллельными пластинами емкость определяется по следующей формуле:

С = ε 0 А / сут

Где C — емкость в Фарадах, ε 0 — постоянная диэлектрической проницаемости свободного пространства (8,85×10 -12), A — площадь пластин в квадратных метрах, а d — расстояние между пластинами в метрах.

Фарада — это очень большая величина емкости, поэтому мы будем использовать метрические префиксы для получения более удобных чисел. Емкость обычно измеряется в микрофарадах (мкФ), что равно 1.0×10 -6F или пикофарад (пФ), что составляет 1,0×10 -12F. 1.0F = 1,000,000 мкФ = 1,000,000,000,000 пФ! Будьте очень внимательны с расчетами!

Назначение:

Целью данной лабораторной работы является исследование взаимосвязи между разделением пластин и емкостью конденсатора с параллельными пластинами.

Оборудование:

  • Конденсатор переменной емкости
  • Цифровой мультиметр
  • Тестер емкости (короткие провода, которые подключаются к мультиметру)
  • Миллиметровая бумага

Осторожно:

Это хрупкое оборудование.Все должно сочетаться с легчайшими прикосновениями. Ничего не заставляйте!

Порядок установки переменного конденсатора

  1. Поместите переменный конденсатор в середину лабораторного стола так, чтобы отметка 0 см находилась слева от вас. Не ставьте конденсатор слишком близко к краю стола!
  2. Поместите мультиметр рядом с пластинами конденсатора. Вам будет удобнее разместить счетчик за конденсатором.
  3. Вставьте тестер емкости в гнездо Cx на мультиметре. По одному выводу тестера входит в каждый из пазов разъема Cx
  4. .
  5. Поместите пластины на расстоянии 5 мм (совместите левый край пластикового выступа, выступающего к шкале, с отметкой 5 мм на шкале). Обратите внимание, что шкала откалибрована в см, поэтому отметка 5 мм будет находиться посередине между 0 см и 1 см.
  6. Присоедините провода измерителя емкости к пластинам конденсатора.Лучше всего их закрепить на радиальных фланцах на задней стороне пластин. Имеются обвязочные столбы, но фланцы работают лучше!


Измеритель и сами провода имеют некоторую емкость (около 4 пФ), поэтому выводы должны быть короткими. Старайтесь держать провода как можно дальше друг от друга.

  1. Поверните большую шкалу мультиметра на «2000p». Включает измеритель и настраивает показания в пикофарадах (пФ).Пикофарад составляет 1,0х10 -12 фарад.

  2. Отведите всех от прибора, дайте измерителю установиться на постоянное значение и запишите показания в столбце «Экспериментальная емкость» в строке 5 мм:

Разделение пластин

(мм)

Экспериментальная емкость

(пф)

Теоретическая емкость

(пф)

Разница в емкости
(пФ)

% Ошибка

5

10

15

20

25

30

35

40

45

50

55

60

65

  1. Повторите эту процедуру, перемещая пластины на 5 мм друг от друга для каждого измерения, пока не достигнете 65 мм.Запишите каждое измерение в таблицу. Когда показание становится меньше 15 пФ, вы можете перейти на шкалу 200 пФ для большей точности.

Анализ данных:

Сначала необходимо рассчитать теоретическую емкость для каждого шага. Мы сделаем первое, а потом вы сможете сделать все остальное! Самая сложная часть этого — правильно настроить юниты. Проще всего поставить все в метрах для расчетов:

  1. Измерьте диаметр пластин конденсатора в сантиметрах.Ваше измерение должно быть около 17,8 см
  2. Разделите диаметр на 100, чтобы получить размер в метрах. Результат — 0,178 м. Разделите это на два, чтобы получить радиус: 0,089 м
  3. Площадь пластины определяется по общей формуле A = πr 2. Подставьте числа, чтобы получить A = π (0,089) 2 = 0,0249 м 2
  4. Преобразуйте расстояние между пластинами (5 мм) в метры, разделив на 1000. 5/1000 = 0,005 м.
  5. Используйте это число в формуле C = ε 0A / d, чтобы определить теоретическую емкость, таким образом: C = 8.85×10 -12 (0,0249) / 0,005 = 4,40×10 -11. Это равно 44.0×10 -12F или 44.0pF
  6. Запишите этот результат (44,0 пФ) в столбец «Теоретическая емкость» и в строку 5 мм.
  7. Повторите этот процесс для других расстояний между пластинами. Обратите внимание, что площадь пластины одинакова для всех, поэтому все, что вам нужно сделать, это повторить шаги 5 и 6, вставляя правильные значения для интервала в каждом случае.
  8. Для каждого шага пластин в таблице найдите разницу между экспериментальным и теоретическим значениями, вычтя каждое теоретическое значение из каждого экспериментального значения.Запишите разницу в таблице.
  9. Теперь вы рассчитаете экспериментальную ошибку для каждого интервала. Просто используйте формулу: E = (теоретическое значение — экспериментальное значение) x100 / теоретическое значение. Например, при настройке 5 мм, если ваше теоретическое значение составляет 44,0 пФ, а экспериментальное значение — 61,1 пФ, вы просто вставляете их в формулу:
    E = (44,0-61,1) x100 / 44,0 = -38,9%. Возьмите абсолютное значение этого числа (38,9%) и запишите его в столбец «% ошибки» таблицы.
  10. На миллиметровой бумаге постройте расстояние между пластинами по оси x (горизонтальная) в зависимости от емкости по оси y (вертикальная). Постройте как теоретическое, так и экспериментальное значение, используя разные цвета или стили линий, чтобы различать две кривые. Убедитесь, что вы выбрали подходящие масштабы и четко обозначили оси и масштабы. Лучше всего ориентировать бумагу длинной осью в горизонтальном направлении («альбомный режим»).
  11. Изучите свой график и ответьте на следующие вопросы:
    1. Подтверждают ли ваши экспериментальные данные теоретические значения?
    2. Вы можете объяснить ошибку эксперимента? (Подсказка: есть ли у измерителя встроенная емкость?)

    3. Какой набор данных более полезен для понимания причин экспериментальной ошибки, разницы между экспериментальными и теоретическими значениями или процента ошибки? Объясните, почему вы так себя чувствуете.
    4. Можно ли изменить номинал конденсатора без изменения размера или расстояния между пластинами? (Подсказка: подумайте, что произошло с показаниями, когда вы были физически близко к конденсатору!)

емкость | Определение, формула, единицы измерения и факты

Емкость , свойство электрического проводника или набора проводников, которое измеряется количеством разделенного электрического заряда, который может храниться на нем на единицу изменения электрического потенциала.Емкость также подразумевает соответствующее хранение электрической энергии. Если электрический заряд передается между двумя первоначально незаряженными проводниками, оба становятся одинаково заряженными, один положительно, другой отрицательно, и между ними устанавливается разность потенциалов. Емкость C — это отношение количества заряда q на любом проводе к разности потенциалов V между проводниками, или просто C = q / В.

Подробнее по этой теме

электричество: Емкость

Полезное устройство для хранения электрической энергии состоит из двух проводов, расположенных в непосредственной близости и изолированных друг от друга.Простой пример …

Как в практических, так и в научных системах метр-килограмм-секунда единицей электрического заряда является кулон, а единицей разности потенциалов является вольт, так что единица емкости — фарад (обозначенная буквой F) — равна единице. кулон на вольт. Один фарад — это чрезвычайно большая емкость. Обычно используются удобные единицы деления на одну миллионную фарада, называемую микрофарад ( μ F), и одну миллионную микрофарада, называемую пикофарадом (пФ; более старый термин, микромикрофарад, μμ F).В электростатической системе единиц емкость имеет размерность расстояния.

Емкость в электрических цепях намеренно вводится устройством, называемым конденсатором. Он был открыт прусским ученым Эвальдом Георгом фон Клейстом в 1745 году и независимо голландским физиком Питером ван Мушенбруком примерно в то же время, когда он исследовал электростатические явления. Они обнаружили, что электричество, полученное от электростатической машины, можно хранить в течение определенного периода времени, а затем высвобождать.Устройство, которое стало известно как лейденская банка, состояло из закрытой пробкой стеклянной пробирки или сосуда, наполненного водой, с гвоздем, протыкающим пробку и погружающимся в воду. Удерживая банку в руке и прикоснувшись гвоздем к проводнику электростатической машины, они обнаружили, что после отсоединения гвоздя можно получить электрический ток, прикоснувшись к нему свободной рукой. Эта реакция показала, что часть электричества от машины была сохранена.

Простой, но фундаментальный шаг в эволюции конденсатора был сделан английским астрономом Джоном Бевисом в 1747 году, когда он заменил воду металлической фольгой, образующей покрытие на внутренней поверхности стекла, и другим, покрывающим внешнюю поверхность.Конденсатор этой формы с проводником, выступающим из горловины сосуда и касающимся футеровки, имел в качестве основных физических характеристик два проводника протяженной площади, почти одинаково разделенных изолирующим или диэлектрическим слоем, сделанным настолько тонким, насколько это практически возможно. Эти особенности сохранены во всех современных конденсаторах.

Получите подписку Britannica Premium и получите доступ к эксклюзивному контенту. Подпишитесь сейчас

Конденсатор, также называемый конденсатором, по существу представляет собой сэндвич из двух пластин из проводящего материала, разделенных изоляционным материалом или диэлектриком.Его основная функция — хранить электрическую энергию. Конденсаторы различаются размером и геометрическим расположением пластин, а также типом используемого диэлектрического материала. Отсюда и такие названия, как слюдяные, бумажные, керамические, воздушные и электролитические конденсаторы. Их емкость может быть фиксированной или регулируемой в диапазоне значений для использования в схемах настройки.

Энергия, запасенная конденсатором, соответствует работе, выполняемой (например, аккумулятором) по созданию противоположных зарядов на двух пластинах при приложенном напряжении.Количество заряда, которое может быть сохранено, зависит от площади пластин, расстояния между ними, диэлектрического материала в пространстве и приложенного напряжения.

Конденсатор, включенный в цепь переменного тока (AC), попеременно заряжается и разряжается каждый полупериод. Таким образом, время, доступное для зарядки или разрядки, зависит от частоты тока, и если требуемое время превышает продолжительность полупериода, поляризация (разделение заряда) не завершается.В таких условиях диэлектрическая проницаемость оказывается меньше, чем наблюдаемая в цепи постоянного тока, и изменяется с частотой, становясь ниже на более высоких частотах. Во время смены полярности пластин заряды должны перемещаться через диэлектрик сначала в одном направлении, а затем в другом, и преодоление сопротивления, с которым они сталкиваются, приводит к выделению тепла, известному как диэлектрические потери, характеристика, которая должна быть учитывается при применении конденсаторов в электрических цепях, например, в радио- и телевизионных приемниках.Диэлектрические потери зависят от частоты и материала диэлектрика.

За исключением утечки (обычно небольшой) через диэлектрик, ток через конденсатор не течет, когда на него действует постоянное напряжение. Однако переменный ток проходит легко и называется током смещения.

Емкость и конденсаторы с примером

Емкость и конденсаторы

Емкость — это отношение накопленного заряда к полученному потенциалу проводников.Единица измерения емкости — кулон на вольт, она называется фарадом (F).

Емкость — это скалярная величина. График, приведенный ниже, показывает соотношение полученного заряда и накопленного потенциала проводящей сферы.

Существует линейная зависимость между полученным зарядом и полученным потенциалом.Наклон графика дает нам емкость сферы.

Как я уже сказал, фарад — это единица емкости, однако мы обычно используем (пФ) пикофарад = 10 -12 Ф, (мкФ) микрофарад = 10 -6 Ф и (нФ) нанофарад = 10 — 9 F.

Сфера радиусом r и зарядом q имеет емкость;

Конденсаторы

Конденсаторы — это устройства, предназначенные для накопления заряда.Они обычно используются в компьютерах или электронных системах. Они состоят из двух проводящих пластин, расположенных на некотором расстоянии друг от друга. Они не касаются друг друга. Когда мы соединяем отрицательно заряженную пластину с нейтральной сферой, они разделяют общий заряд до тех пор, пока потенциалы не сравняются и лепестки электроскопа не поднимутся. Затем мы размещаем пластину A на расстоянии d от B. Поскольку мы заземляем пластину, она вначале нейтральна. Поскольку B заряжен отрицательно, он воздействует на пластину A и заряжается положительно за счет индукции.Если положить между пластинами другой изолятор, например пластик, то створки электроскопа немного сомкнуты. Можно сделать вывод, что емкость пластин зависит от расстояния между пластинами.

В схеме изображаем конденсатор символом;

А батарея, которая питает разность потенциалов, обозначена символом;

Мы показываем конденсаторы и батарею в схеме, как показано ниже;


Емкость пластины зависит от;

· Площадь плит

· Расстояние между пластинами d

· Диэлектрическая проницаемость между пластинами ε º

Емкость пластин определяется по следующей формуле;


Диэлектрическая проницаемость между пластинами ε º зависит от типа материала.Например, вакуум имеет ε = 8, 85,10 -12 Ф / м, а вода имеет ε = 717,10 -12 Ф / м.

Пример: Рассчитайте емкость конденсатора, имеющего размеры 30 см X 40 см и разделенного воздушным зазором d = 8 мм.

A = 30,10 -2 м X 40,10 -2 м = 0,12 м 2

C = (8, 85,10 -12 C 2 / Н.м 2 ) .0,12 м 2 / 8,10 -3 м

C = 0, 13275.10 -9 Ф

Электростатические экзамены и решения

Электрический потенциал и электрическая потенциальная энергия <Пред. Далее> Конденсаторы последовательно и параллельно

Формулы и калькуляторы емкости

На этой странице представлены формулы и калькуляторы емкостей различные формы или типы конденсаторов. Это также полезно, если вы собираетесь использовать свой конденсатор в Танк LC резонансный схема.

Емкость конденсаторов с параллельными пластинами

Конденсатор с параллельными пластинами состоит из двух плоских параллельных пластин, которые электроды, разделенные диэлектрик или изолятор. Для формулы и калькулятора здесь пластины могут быть любой формы, если они плоские, параллельные и вы знаете площадь тарелки или что-то еще, что нужно, чтобы найти этот район.

Конденсатор с параллельными пластинами — пластины прямоугольной формы.
Конденсатор с параллельными пластинами — круглые пластины.

Формула емкости конденсатора с параллельными пластинами:

Где:

  • ε r = относительная диэлектрическая проницаемость диэлектрика (реже К, диэлектрическая проницаемость)
  • ε 0 = 8.854×10 -12 Ф / м (фарад / метр) = диэлектрическая проницаемость вакуума или диэлектрическая проницаемость свободного пространства

На схемах показаны конденсаторы с параллельными пластинами разной формы. пластины, одна прямоугольная и одна круглая. Формула для расчета площадь прямоугольника:

а формула для вычисления площади круга:

Где π — это число пи, равное 3,14159.

Емкость цилиндрических конденсаторов

Цилиндрический конденсатор состоит из двух цилиндров, также называемых пластины, которые являются электродами, разделены диэлектрик или изолятор.

Цилиндровый конденсатор.

Формула емкости цилиндрического конденсатора:

Где:

  • ε r = относительная диэлектрическая проницаемость диэлектрика (реже К, диэлектрическая проницаемость)
  • ε 0 = 8,854×10 -12 Ф / м (фарад / метр) = диэлектрическая проницаемость вакуума или диэлектрическая проницаемость свободного пространства

Видео — Как сделать конденсаторы — Низкое напряжение

В этом видео не только показано, как делать конденсаторы, но и формула емкости в более динамичном формате, чем указано выше.После всего, если вы делаете конденсатор, вам сначала нужно знать, как спроектировать конденсатор.

Видео — Как сделать конденсаторы — Высокое напряжение

В этом видео показано, как разработать конденсаторы для высокого напряжения, объясняя, измерения и построения для напряжения пробоя / диэлектрической прочности, чтобы что конденсатор может выдерживать желаемое высокое напряжение.

.

Добавить комментарий

Ваш адрес email не будет опубликован. Обязательные поля помечены *